Температура воздуха в комнате изменилась от 7 до 27 на сколько: Температура воздуха в комнате изменилась от 7 до 27 °С. На сколько процентов уменьшилось число молекул в комнате?

\circ\) C, \(\frac{\Delta N}{N_0}-?\)

Содержание

Решение задачи:

Для решения этой задачи воспользуемся следующей формулой определения давления:

\[p = nkT\]

В этой формуле \(n\) – концентрация молекул, а \(k\) – постоянная Больцмана.

Концентрация молекул – это отношение числа молекул к объему, занимаемому ими:

\[n = \frac{N}{V}\]

В условии температура дана в градусах Цельсия, а в нашей формуле для определения давления ее необходимо  указывать в Кельвинах, поэтому воспользуемся формулой связи этих шкал:

\[T = t + 273\]

Запишем формулы для определения давления в двух случаях:

\[p = \frac{{{N_0}}}{V}k\left( {{t_0} + 273} \right)\]

\[p = \frac{{{N_1}}}{V}k\left( {{t_1} + 273} \right)\]

Так как давление не меняется (\(p=const\)), то приравняется эти два выражения.

\[\frac{{{N_0}}}{V}k\left( {{t_0} + 273} \right) = \frac{{{N_1}}}{V}k\left( {{t_1} + 273} \right)\]

\[{N_0}\left( {{t_0} + 273} \right) = {N_1}\left( {{t_1} + 273} \right)\]

\[\frac{{{N_1}}}{{{N_0}}} = \frac{{{t_0} + 273}}{{{t_1} + 273}}\]

В условии сказано, что необходимо найти как изменится число молекул в комнате, т.е. нужно найти отношение \(\frac{\Delta N}{N_0}\).

\[\frac{{\Delta N}}{{{N_0}}} = \frac{{{N_1} – {N_0}}}{{{N_0}}} = \frac{{{N_1}}}{{{N_0}}} – 1\]

В итоге:

\[\frac{{\Delta N}}{{{N_0}}} = \frac{{{t_0} + 273}}{{{t_1} + 273}} – 1\]

\[\frac{{\Delta N}}{{{N_0}}} = \frac{{17 + 273}}{{27 + 273}} – 1 =  – 0,033 =  – 3,3\% \]

Знак “минус” показывает, что количество молекул уменьшится.

Ответ: уменьшится на 3,3%

Если Вы не поняли решение и у Вас есть какой-то вопрос или Вы нашли ошибку, то смело оставляйте ниже комментарий.

Если Вам понравилась задача и ее решение, то Вы можете поделитесь ею с друзьями с помощью этих кнопок.

Оглавление

Глава 2


Молекулярно-кинетическая теория

2.1 Строение вещества. Уравнение состояния

2.1.1 Пример – количество атомов

Плотность алюминия 2,7 ⋅ 103 кг/м3. Сколько атомов алюминия содержится в объеме 1 см3? Молекулярная масса алюминия 27 ⋅ 10-3 кг/моль.

Решение

Один моль (27 г) алюминия содержит 6 ⋅ 1023атомов. В 1 см3содержится 2,7 г, то есть в 10 раз меньше.

Ответ: 6 ⋅ 1022атомов.

2.1.2 Пример – химический состав

Найдите формулу соединения азота с кислородом, если его масса m = 1 г в газообразном состоянии в объеме V = 1 л создает при температуре T = 17C давление P = 3,17 ⋅ 104 Па.

Решение

Давление определяется из уравнения состояния идеального газа

Определим молекулярную массу соединения Подставим числа, выразив их в системе единиц СИ: m = 1 г = 10-3 кг, R = 8,31 Дж/(моль⋅К), V = 1 л = 10-3 м3, P = 3,17 ⋅ 104 Па, T = 290 К. Отсюда Пусть в соединении содержится x атомов азота и y атомов кислорода. Молекулярная масса азота 14 г/моль, кислорода. 16 г/моль. Тогда должно выполняться равенство: Так как x и y — целые числа, легко найти, что x = 2,y = 3.

Ответ: соединение N2O3.

2.1.3 Пример – воздух в комнате

Температура воздуха в комнате объемом V = 45 м3повысилась от t1= 17C до t2= 27C. На сколько уменьшилась масса воздуха в комнате? Атмосферное давление P0= 105 Па постоянно. Молекулярная масса воздуха μ = 29 г/моль.

Решение

Пусть m1масса воздуха, соответствующая начальной температуре T1= 290 K, а m2– конечной T2= 300 K. Запишем уравнения состояния газа для начального и конечного состояний:

Искомая масса воздуха Δm = m1— m2.

Выражая массы из первых двух уравнений и подставляя в третье, получим

Подставив числа, получим ответ Δm = 1,8 кг.
2.1.4 Пример – воздушный шар

Во сколько раз изменится подъемная сила воздушного шара, если наполняющий его гелий заменить водородом? Весом оболочки шара пренебречь.

Решение

Пусть атмосферное давление P, температура T, объем воздушного шара V . Масса воздуха, содержащаяся при этих условиях в объеме V , определится из уравнения состояния

где μ0– молекулярная масса воздуха. Обозначим через μ1молекулярную массу водорода, а через μ2– молекулярную массу гелия. Тогда массы водорода m1и гелия m2, содержащихся в объеме V , выразятся соотношениями Соответствующие подъемные силы определятся разницей веса воздуха и газа в объеме V : Подставив в последние уравнения выражения масс, получим для отношения подъемных сил Подставляя численные значения μ0= 29 г/моль, μ1= 2 г/моль, μ2= 4 г/моль, получим

2.2 Термодинамика

2.2.1 Пример – давление и энергия идеального газа

Одноатомный идеальный газ изотермически расширяется из состояния с давлением P = 10

5Па от объема V 1= 1 м3до объема V 2= 5 м3. Какова внутренняя энергия и давление газа в конечном состоянии?

Решение

Внутренняя энергия идеального газа при изотермическом расширении не меняется. Поэтому ее можно найти из начального состояния. Для одноатомного газа

И из данных задачи получаем При постоянной температуре P1V 1= P2V 2. Откуда конечное давление
2.2.2 Пример – работа, тепло и внутренняя энергия

Газ в цилиндре получил тепло Q = 1000 Дж и, расширившись, совершил работу A = 300 Дж. На сколько изменилась при этом его внутренняя энергия?

Решение

Согласно первому закону термодинамики изменение внутренней энергии газа равно количеству тепла, полученного газом, минус работа, совершенная газом: ΔE = Q — ΔA.

Откуда ΔE = 1000 — 300 = 700 Дж.

2.2.3 Пример – работа и КПД цикла

Один моль одноатомного идеального газа участвует в циклическом процессе, график которого, состоящий из двух изохор и двух изобар, представлен на рисунке 2.1. Температуры в точках 1 и 3 равны T

1и T3. Известно, что точки 2 и 4 лежат на одной изотерме. Определите работу, совершенную газом за цикл и КПД цикла.




Решение

Обозначим через V 1минимальный объем, занимаемый газом (на изохоре 1,2), через V 2– максимальный (на изохоре 3,4). Соответственно, через P1– минимальное (на изобаре 4,1), а через P2– максимальное (на изобаре 2,3) давление. Пусть температура в точках 2 и 3 равна T2.

Работа газа за цикл определится как площадь на диаграмме:

(2.1)

Из уравнения состояния идеального газа для одного моля имеем:

Подставляя (2.2) – (2.5) в (2.1), получим Теперь необходимо найти T2. Для этого разделим почленно (2.2) на (2.3), а (2.4) на (2.5): Приравнивая правые части Откуда Работа газа определится через заданные температуры Найдем теперь КПД цикла. Для этого надо найти тепло, переданное газу за цикл – Q. Газ получает от нагревателя тепло на участках 1,2 и 2,3. На остальных участках газ отдает тепло холодильнику.

При изохорическом процессе тепло, полученное газом:

Для одного моля одноатомного идеального газа cV = 3R∕2. Тогда При изобарическом расширении газ получает тепло где cP= cV + R = 3R∕2 + R = 5R∕2. То есть Полное количество тепла, полученного газом КПД цикла Пусть T3= 900 K, T1= 400 K. Тогда Заметим, что КПД обратимой машины, работающей с нагревателем температуры T3= 900 K, и холодильником с T1= 400 K определится из формулы
2.2.4 Пример – динамическое отопление

С помощью электрической плитки мощностью N = 1 кВт в комнате поддерживается температура воздуха t1= 17C при температуре наружного воздуха t2= -23

∘C. Какая мощность потребовалась бы для поддержания в комнате той же температуры с помощью обратимой тепловой машины?

Решение

Для поддержания нужной температуры запустим тепловую машину по обратному циклу. Тогда наружный воздух будет играть роль нагревателя при T2= 250 K, а воздух в комнате – роль холодильника при T1= 290 K. Процесс пойдет таким образом: забрав тепло Q2от наружного воздуха и затратив некоторую работу A (например, с помощью электродвигателя), передадим тепло Q1внутрь комнаты. Пусть A, Q1и Q2относятся к единице времени (секунде). Тогда для поддержания той же температуры необходимо, чтобы переданное тепло Q1равнялось теплу, поступающему в первом случае:

КПД тепловой машины Так как машина обратимая Приравнивая правые части последних выражений, получим Мощность, которую необходимо затратить Подставляя Q1= 103Вт и температуры, получим При таком способе отопления затрачивается значительно меньше энергии! Вместо электрической мощности 1 кВт можно затрачивать для работы электродвигателя лишь 137 Вт. Такое отопление называется динамическим. Идею использования динамического отопления выдвинул лорд Кельвин.

2.3 Фазовые переходы. Поверхностное натяжение

2.3.1 Пример – нагревание и кипение воды

В кастрюлю налили воду при температуре t0= 10C и поставили на плиту. Через τ = 10 минут вода закипела. Через какое время она полностью превратится в пар?

Решение

Обозначим через c удельную теплоемкость воды. Будем считать, что теплоемкость при росте температуры не меняется: c = 1 кал/(г⋅град) = 4,18 ⋅ 103 Дж/(кг⋅град).

Пренебрежем потерями тепла, считая, что за единицу времени при любой температуре воде передается одинаковая энергия w. Пусть масса воды m. Тогда за время τ вода получила энергию wτ, которая пошла на нагревание до температуры кипения t = 100C:

(2.1)

Для того, чтобы вода выкипела, она должна получить энергию mq, где q = 2,26 ⋅ 10

6Дж/кг – удельная теплота парообразования воды. Такое тепло вода получит за время τ1:

(2.2)

Выразив массу из (2.1) и подставив в (2.2) получаем

Откуда Подставляя числа

Ответ: вода полностью выкипит через час после закипания.

2.3.2 Пример – пограничное кипение

В стакан налиты две несмешивающихся жидкости: четыреххлористый углерод (CCl4) и вода. При нормальном атмосферном давлении CCl4кипит при 76,7C, вода – при 100C. При равномерном нагревании стакана со смесью в водяной бане кипение на границе раздела жидкостей начинается при температуре 65,5С. Определите, какая из жидкостей быстрее (по массе) выкипает при таком пограничном кипении и во сколько раз. Давление насыщенного пара воды при 65,5С составляет 25,6 кПа.

Решение

При пограничном кипении газовые пузырьки растут на границе раздела жидкостей. При этом сумма парциальных давлений CCl

4(P1) и воды (P2= 25,6 кПа) должна равняться атмосферному давлению (при нормальных условиях P0= 1 атм = 100 кПа):

(2.1)

Масса m1четыреххлористого углерода, находящегося в некотором пузырьке объема V , определится из уравнения Клапейрона-Менделеева:

(2.2)

Здесь μ1= 154 г/моль – молекулярная масса CCl4, T = 338,5 K – абсолютная температура, соответствующая 65,5C. Аналогично для воды

(2.3)

где μ2= 18 г/моль.

Разделив почленно (2.2) на (2.3), имеем

(2.4)

Подставив в (2.4) давление P1, выраженное из (2.1), получим

Подставив числа, получим ответ
2.3.3 Пример – влажность воздуха

При одинаковой температуре смешали (объединив объемы) V

1= 1 м3воздуха влажностью φ1= 20% и V 2= 2 м3воздуха влажностью φ2= 30%. Определите относительную влажность смеси.

Решение

Обозначим давление насыщенного пара при температуре T через P0, парциальное давление водяного пара массы m1в объеме V 1= 1 м3– через P1, а водяного пара массы m2в объеме V 2= 2 м3– через P2.

По определению влажность (выраженная не в процентах) в первом объеме

(2.1)

а во втором

(2.2)

Параметры водяного пара в каждом случае определятся из уравнения Клапейрона-Менделеева

После объединения объемов установится парциальное давление водяного пара P, определяемое из уравнения
(2.5)

Выразив из (2.1) и (2.2) P1= φ

1P0и P2= φ2P0, а также m1и m2из (2.3) и (2.4) и подставив все в (2.5), получим

Относительная влажность воздуха определится как Подставляя числа Так как относительная влажность воздуха выражается в процентах, ответ φ ≈ 27%.
2.3.4 Пример – капля ртути

Какую работу против сил поверхностного натяжения нужно совершить, чтобы разделить сферическую каплю ртути радиуса r0= 3 мм на две одинаковые капли? Коэффициент поверхностного натяжения ртути σ = 0,465 Н/м.

Решение

Работа против сил поверхностного натяжения определяется изменением полной поверхности

(2.1)

Начальная поверхность S0= 4πr02. Обозначим радиус каждой из двух образовавшихся капель через r. Тогда полная поверхность двух капель S = 2 ⋅ 4πr2. Так как ΔS = S — S0, работа

(2.2)

При разделении капли сохраняется полный объем ртути

(2.3)

Откуда находится радиус образовавшихся капель. Подставив r из (2.3) в (2.2), получим

Подставляя числа, получим ответ

Основное уравнение молекулярно-кинетической теории

Урок № 6.

Тема урока. Основное уравнение молекулярно-кинетической теории

Цель: обучающая — способствовать более глубокому пониманию понятий, характеризующих основное уравнение МКТ, совершенствовать навыки решения задач.

развивающая: формировать у обучающихся умение осуществлять самоконтроль с помощью конкретных вопросов, совершенствовать навыки индивидуальной и групповой работы; активизировать мышление школьников, умение самостоятельно формулировать выводы, развивать речь.

воспитательная: развить чувства взаимопонимания и взаимопомощи в процессе изучения предмета; развивать мотивацию изучения физики; подтвердить факт необходимости знания МКТ для решения технических задач.

Тип урока: закрепления знаний, умений, навыков.

Методы обучения: работа в группах, в парах.

План урока

1. Орг.момент.

2. Актуализация опорных знаний.

3. Решение задач в группах, парах.

4. Самостоятельная работа.

5. Домашнее задание.

Ход урока

  1. Орг.момент. 

  1. Актуализация опорных знаний.

1. Сформировать и записать на доске основное уравнение МКТ устанавливающее связь между давлением молекул на стенки сосуда и микроскопическими параметрами газа:

2. Записать на доске уравнение давления газа на стенки сосуда через среднюю кинетическую энергию поступательного движения молекул:

3. Выразить давление идеального газа на доске через плотность газа и средний квадрат скорости поступательного движения молекул:

4. На доске написано слева и справа частицы формул в два столбика. Установить соответствие между этими частицами так, чтобы образовалась правильно записанная формула. На выполнение этого задания отводится 2 минуты.

Через 2 минуты учитель предоставляет слово учащимся и устанавливает критерии оценок. За правильный ответ:

• 5 — ученик получает 11 баллов,

• 4 — ученик получает 9 баллов,

• 3 — ученик получает 6 баллов,

• 2 — ученик получает 4 балла,

• 1 — ученик получает 2 балла.

3. Решение задач в группах, парах.

Задача 1. Плотность газа в баллоне электрической лампы ρ = 0,9 кг/м2. При горении лампы давление в ней возросло с р1 = 8 ·104 Па до р2 = 1,1· 105 Па. На сколько увеличилось при этом значение среднего квадрата скорости молекул газа?

Решение. Произведение массы одной молекулы на концентрацию молекул (число молекул в единице объёма) равно массе молекул, заключённых в единице объёма, т. е. плотности газа. Следовательно, основное уравнение молекулярно-кинетической теории можно записать в виде 

Поэтому 

Задача 2. Определите плотность кислорода ρO при давлении 2·105 Па, если средний квадрат скорости его молекул равен 106(м/с)2.

Решение. Давление кислорода  где n — концентрация молекул. Очевидно, что ρ = ·n, где  — масса молекулы кислорода.

Окончательно имеем 

Задача 3. Два одинаковых сосуда, содержащие одинаковое число молекул азота, соединены краном. В первом сосуде средний квадрат скорости молекул  во втором сосуде —  Кран открывают. Чему будет равен средний квадрат скорости молекул после того, как установится равновесие?

Решение. Разные скорости молекул в сосудах объясняются разными температурами азота в них. Так как по условию задачи число молекул, имеющих скорость υ1, равно числу молекул, имеющих скорость υ2 (N1 = N2), то квадрат средней скорости

Задача 4. С какой скоростью растёт толщина покрытия стенки серебром при напылении, если атомы серебра, обладая энергией , производят на стенку давление р = 0,1 Па? Атомная масса серебра А = 1,108 г/моль, его плотность ρ = 10,5 г/см3.

Решение. Если за время Δt толщина слоя серебра стала равной Δl, то скорость роста толщины покрытия есть Δl/Δt. Объём напылённого слоя ΔV = SΔl, где S — площадь поверхности стенки. Этот объём можно выразить иначе:

где m — масса серебряного покрытия, напылённого за время Δt, — масса атома, N — число атомов. Определим суммарную массу атомов серебра, осевших на стенку.

Изменение импульса атома, осевшего на стенку со скоростью υ, равно импульсу силы, подействовавшей на стенку со стороны атома:

ƒτ = Δυ = (0 — υ) = -υ.

На стенку подействует импульс силы ƒстτ = +υ. Если на стенку за время Δt осядет N атомов, то импульс силы, подействовавший на стенку в результате ударов о неё N атомов, будет FΔt = Nυ.

Давление на стенку р = F/S, или

р = Nυ/SΔt.                     (1)

Средняя кинетическая энергия атома  отсюда скорость атома 

Подставив выражение для скорости в формулу (1), получим

тогда

Масса атома серебра = A/NA, где NA = 6,02 • 1023 моль-1. Подставив это выражение в формулу (2), получим

Задача 5. Определить массу водорода, находящегося в баллоне вместимостью 20 л при давлении 830 кПа, если температура газа равна 17 °С.

Решение:

Выведем размерность искомой физической величины:

Ответ: m = 1,38· кг.

Задача 6. В баллоне находится газ под давлением 40 Па и при температуре 27°С. Когда из баллона выпустили 3/5 газа, содержавшегося в нем, его температура понизилась до -33 °С. Определить давление газа, оставшегося в сосуде.

Решение:

Ответ: = 12,8 Па.

Задача 7. Сколько молекул воздуха выходит из комнаты объемом 120 , если температура повышается от 15 до 25°С? Атмосферное давление равно 105 Па.

Решение:

 = 6· — число Авогадро;

R = 8,31 Дж/(моль·К) — универсальная газовая постоянная;

Значение Δm подставляем в уравнение (1):

Выведем размерность искомой физической величины:

Ответ: N = 1· молекул.

 

  1. Самостоятельная работа.

Задачи для самостоятельного решения

  1. Температура воздуха в комнате изменилась от 7 до 27 °С. На сколько процентов уменьшилось число молекул в комнате?

  1. Под каким давлением находится газ в сосуде, если средний квадрат скорости его молекул  концентрация молекул n = 3·1025 м-3, масса каждой молекулы  = 5 ·кг?

  1. В колбе объёмом 1,2 л содержится 3·1022 атомов гелия. Чему равна средняя кинетическая энергия каждого атома? Давление газа в колбе 105 Па.

  1. Вычислите средний квадрат скорости движения молекул газа, если его масса m = 6 кг, объём V = 4,9 м3 и давление р = 200 кПа.

5. Домашнее задание: 

  1. Г.Я.Мякишев, Б.Б.Буховцев, Н.Н.Сотский. Физика, 10 класс, М., «Просвещение», 2016. Повторить §57 (с.190-192).

  2. Решить задачи по вариантам (2 варианта):

2.1. В сосуде содержится 10 г газа под давлением 680 мм. рт. ст. Средняя квадратичная скорость молекул 230 м/c. Найти объем сосуда.

2.2. Средняя квадратичная скорость молекул газа, находящегося при температуре 100 °С равна 540 м/c. Определить массу молекулы газа?

2.3. Чему равняется отношение произведения давления газа на его объем к числу молекул при температуре t = 300 °С.

2.4. Современные вакуумные насосы позволяют снизить давление до 1,3·10-10 Па (10-12 мм рт. ст.). Сколько молекул газа содержится в 1 см3 при определенных давлении и температуре 27 °С?

3. Дополнительное задание.

Баллон объемом 100 л содержит гелий под давлением  = 5· Па при температуре  = 300 К. Вентиль баллона испортился и гелий начал вытекать. Когда отремонтировали вентиль, давление в баллоне оказался равным = 5,1· Па, но температура повысилась до = 340 К. Найти массу гелия, которая вытекла из баллона. Молярная масса гелия М = 4·кг/моль.

Тема №5093 Решение задач по физике молекулярная физика

Тема №5093

Ответы в самом низу встроенного документа

9.1.    На фотоснимке видимый диаметр молекулы некоторого вещества равен 0,5 мм. Чему равен действительный диаметр молекулы данного вещества, если фотоснимок получен с помощью электронного микроскопа с увеличением в 200 000 раз?
9.2.    Капля масла объемом 0,003 мм3 растеклась по поверхности воды тонким слоем и заняла площадь 300 см2. Принимая толщину слоя равной диаметру молекулы масла, определить этот диаметр.
9.3.    Можно ли сказать, что объем газа в сосуде равен сумме объемов его молекул?
9.4.    Если смешать по два равных объема ртути и воды, спирта и воды, то в первом случае получится удвоенный объем смеси, а во вто-ром — меньше удвоенного объема. Почему?
9.5.    Чем отличается траектория движения молекулы в воздухе от ее траектории в вакууме?
9.6.    Броуновские частицы в опытах Перрена имели размер 1 мкм. Во сколько раз они больше молекул воды, диаметр которых 10“8 см?
9.7.    Какое количество вещества содержится в медном бруске массой 6 кг?
9.8.    В сосуде находится 5,418 * 10е молекул кислорода. Какое количество вещества, выраженное в молях, находится в этом сосуде?
9.9.    Какова масса 200 молей азота?

9.10.    Капелька воды имеет массу 10 10 г. Из скольких молекул она состоит?
9.11.    Какое количество молекул содержится в т = 10 г водорода?
9.12.    Из открытого стакана за время t — 5 сут полностью испарилось т = 50 г воды. Сколько в среднем молекул вылетало с поверхности воды за одну секунду?
9.13.    Плотность воды при нормальных условиях р = 103 кг/м3. Можно считать, что каждая молекула воды ограничена сферой радиуса R. Эти сферы касаются друг друга, а их центры образуют кубическую решетку. Оценить массу молекулы воды и ее размер.
9.14.    Подсчитать число молекул, содержащееся в углекислом газе массой т = 100 г. Найти массу молекулы и концентрацию молекул при нормальных условиях. Плотность газа при нормальных условиях р = 1,94 кг/м3.
9.15.    Оценить концентрацию свободных электронов в натрии, полагая, что на один атом приходится один свободный электрон. Плотность натрия р = 970 кг/м3.
9.16.    Вычислить плотность водорода, если известно, что число его молекул N в сосуде вдвое больше числа Авогадро iVA, а объем сосуда
V    = 40 л.
9.17.    Какое количество вещества содержится в кислороде массой т = 48 г, если треть его молекул находится в диссоциированном состоянии?
9.18.    Определить расстояние между ближайшими ионами кубической кристаллической решетки железа.
9.19.    Принимая молекулу газа за шарик диаметром d — 0,3 нм, вычислить, какую часть объема газа при нормальных условиях занимают его молекулы. Каково среднее расстояние между ними?
9.20.    Плотность неизвестного газа р = 0,09 кг/м3. При этом в объеме
V    = ОД м3 содержится N = 2,7 * 1024 молекул. Какой это газ? Определить его молярную массу.
9.21.    Радоновые ванны содержат N = 1,8 • 106 атомов радона на объем воды V — 1 дм3. На сколько молекул воды приходится один атом радона?
9.22.    Какая масса т углекислого газа С02 растворена в бутылке с лимонадом объемом V = 0,5 л, если на одну молекулу газа приходится
= 5,56 * 105 молекул воды?
3800 задач по физике для школьников и поступающих в вузы 1ST
9.    Молекулярно-кинетическая теория. Уравнение состояния
9.23.    Если пометить все молекулы в одном стакане воды и вылить эту воду в Мировой океан, а потом вновь зачерпнуть стакан воды, то сколько в нем будет меченых молекул? Объем воды Мирового океана Vx = 1,31 • 1018 м3, объем стакана V2 = 200 см3.
9.24.    Сравнить число молекул воды и ртути, содержащееся в равных объемах.

9.25.    Каково давление газа, если в каждом кубическом сантиметре его содержится п = 10е молекул, а температура газа t = 87 °С?
9.26.    Давление газа в современной телевизионной трубке р ~ 10“9 атм. Какое число молекул содержится в объеме V = 1 см3 при комнатной тем- пературе?
9.27.    Современные вакуумные насосы позволяют понижать давление до р — 10-12 мм рт. ст. Сколько молекул газа содержится в объеме V = 1 см3 при этом давлении и температуре t = 48 °С?
9.28.    Сколько молекул газа находится в сосуде объемом V = 480 см3 при температуре t = 20 °С и давлении р = 250 кПа?
9.29.    Оценить число молекул воздуха в атмосфере Земли, считая, что молярная масса воздуха М = 29 ■ 10~3 кг/моль.
9.30.    Оценить среднее расстояние между молекулами газа при нор-мальных условиях.
9.31.    Во сколько раз изменится среднее расстояние между молекулами газа, если при постоянном давлении повысить температуру в 4 раза?
9.32.    Какое давление на стенки сосуда производит кислород, если скорость его молекул v = 400 м/с и в объеме V = 1 см3 число молекул N = 2,7 • 1019?
9.33.    В закрытом сосуде находится один моль некоторого идеального газа. Из опытов найдено, что £ — 371 Па/К. Определить концентрацию молекул и объем сосуда.
9.34.    В сосуде объемом V = 10 л находится N = 1,24 * 1Q24 молекул кислорода 02 при давлении р = 4 атм. Какое количество атомов гелия будет находиться в таком же сосуде при тех же давлении и температуре? Чему равна температура газа Т в сосуде?
    
9.35.    Спутник площадью поперечного сечения S = 3 м2 движется по круговой орбите над Землей со скоростью v = 7,9 км/с. Давление воздуха на высоте орбиты р = 1,38 * 10-4 Па, температура Т = 120 К. Определить число столкновений молекул воздуха со спутником за время t = 1 с.
9.36.    Газ плотностью р = 5,95 кг/м3 находится при температуре t — 0 °С. Найти давление газа, если масса молекулы т0 = 7,3 • 10-26 кг. Какой это газ?
9.37.    В баллоне объемом V = 10 л находится газ при температуре t = 27 °С. Вследствие утечки газа давление снизилось на р = 4,2 кПа. Сколько молекул вышло из баллона?
9.38.    Определить среднюю кинетическую энергию поступательного движения молекулы идеального газа, если при давлении р = 2 * 105 Па концентрация молекул газа п = 5 * 1025 м_3.
9.39.    Средняя энергия молекулы одноатомного идеального газа Ек = 0,038 эВ. (1 эВ = 1,6 * 10“19 Дж.) Давление газа р = 0,2 мПа. Найти число молекул в одном кубическом метре газа.
9.40.    При какой температуре средняя кинетическая энергия молекулы идеального одноатомного газа Ек = 2 * 10“2° Дж?
9.41.    Во сколько раз изменится температура аргона, если средняя кинетическая энергия его атома увеличится на г\ = 50% ?
9.42.    На сколько изменится средняя кинетическая энергия атома неона, если его температура уменьшится в п = 4 раза? Начальное значение средней кинетической энергии атома JSKl = 7 • 10″21 Дж.
9.43.    На сколько изменилась температура аргона, если средняя ки-нетическая энергия атома аргона уменьшилась в п = 1,2 раза? Начальная температура аргона Тг = 400 К.
9.44.    Во сколько раз изменится средняя кинетическая энергия атома неона при уменьшении его абсолютной температуры на г| = 30% ?
9.45.    Определить кинетическую энергию N — 105 атомов гелия при температуре t = 47 °С.
9.46.    Гелий находится при температуре t = 27 °С. Кинетическая энергия теплового движения всех молекул газа Ек = 10 Дж. Определить число молекул гелия.
9.47.    В баллоне емкостью V = 50 л находится одноатомный газ в количестве v = 0,12 кмоль при давлении р — 6 • 10е Па. Определить среднюю энергию теплового движения молекулы газа.

9.48.    Газ занимает объем V = 4 л при давлении р = 5 • 102 Па. Найти суммарную кинетическую энергию поступательного движения молекул.
9.49.    При какой температуре средняя энергия теплового движения атома неона будет достаточна для того, чтобы атом преодолел земное тяготение и покинул атмосферу?
9.50.    Во сколько раз изменится давление одноатомного газа в результате уменьшения его объема в 3 раза и увеличения средней кинетической энергии его молекул в 2 раза?

9.51.    Определить среднюю квадратичную скорость молекул кислорода при температуре t = 20 °С. При какой температуре эта скорость будет v = 500 м/с?
9.52.    В воздухе взвешена пылинка массой m = 1,242 • Ю-20 кг. Тем-пература воздуха Т = 300 К. Подсчитать среднюю квадратичную скорость пылинки и ее кинетическую энергию.
9.53.    Капелька воды, взвешенная в воздухе, движется со средней квадратичной скоростью v ~ 1,7 м/с. Радиус капли г = 10“6 см. Найти температуру воздуха.
9.54.    Найти импульс молекулы азота, скорость которой равна средней квадратичной скорости при температуре Т = 300 К.
9.55.    Молекула кислорода, ударившись о стенку сосуда, передала ей импульс Ар = 5,06 • 10“23 кг ■ м/с. Найти температуру газа в сосуде, если скорость данной молекулы была направлена под углом а = 30° к стенке и равнялась по величине удвоенной среднеквадратичной скорости.
9.56.    Определить, во сколько раз средняя квадратичная скорость пы-линки массой m = 1,75 • 1012 кг, взвешенной в воздухе, меньше средней квадратичной скорости движения молекул воздуха.
9.57.    При некоторой температуре молекулы кислорода имеют среднюю квадратичную скорость 460 м/с. Какова при этой температуре средняя квадратичная скорость молекул азота?
9.58.    Средняя квадратичная скорость молекул некоторого газа v — 480 м/с при температуре Т = 296 К. Сколько молекул содержится в m = 10 г этого газа?
9.59.    Повышение температуры газа на АТ = 150 К привело к увеличению средней квадратичной скорости его молекул от vt = 400 м/с до и2 = 500 м/с. На сколько должна измениться температура этого газа, чтобы дополнительно повысить среднюю квадратичную скорость еще на 100 м/с (т. е. увеличить ее с v2 — 500 м/с до о3 = 600 м/с)?

9.60.    Один сосуд заполнен гелием, а другой такой же — кислородом. Температура газов одинакова: Т = 300 К. На сколько следует изменить температуру газа в одном из сосудов, чтобы средние квадратичные скорости молекул гелия и кислорода стали равными? Рассмотреть возможные варианты решения.
9.61.    Во сколько раз возрастет средняя квадратичная скорость и теп-лового движения молекул одноатомного газа, помещенного в закрытый сосуд, движущийся со скоростью и = 2v, при его резкой остановке? Во сколько раз изменится температура газа?
9.62.    Две одинаковые колбы с одинаковым количеством молекул водо-рода в них соединены трубкой с краном. Средняя квадратичная скорость молекул в первой колбе vx = 400 м/с, а во второй — и2 — 600 м/с. Какая установится средняя квадратичная скорость, если открыть кран, соеди-няющий колбы? Теплообмена с окружающей средой нет.
9.63.    Давление газа в закрытом сосуде увеличилось после его нагревания в 16 раз (р2 = 16pj). Во сколько раз изменилась средняя квадратичная скорость его молекул?
9.64.    Сравнить давление кислорода и водорода при одинаковых кон-центрациях молекул, если их средние квадратичные скорости одинаковы.
9.65.    Во сколько раз изменится давление газа при уменьшении его объема в 3 раза, если средняя квадратичная скорость останется неизменной?
9.66.    В закрытом сосуде находится идеальный газ. На сколько процентов изменится его давление, если средняя квадратичная скорость его молекул увеличится на ц = 20% ?
9.67.    Найти концентрацию молекул кислорода, если его давление р = 0,2 МПа, а средняя квадратичная скорость молекул и = 700 м/с.
9.68.    Найти давление электронного газа в германиевом полупроводнике, если известно, что в объеме полупроводника V = 1 см3 содержится N = 1015 свободных электронов, движущихся со средней квадратичной скоростью v = 100 км/с. Электронный газ считать идеальным.
9.69.    При давлении р = 105 Па плотность воздуха р = 1,29 кг/м3. Вычислить среднюю квадратичную скорость его молекул.1? Расстояние между внутренним и внешним цилиндрами Я = 2 см,
9.75.    Каково будет смещение напыленной полоски металла в приборе Штерна при частоте вращения v = 20 с»1 и при скорости атомов и = 300 м/с? Радиус цилиндра R — 10 см.
9.76.    Два баллона соединены непроводящей тепло тонкой трубкой. Объемы баллонов Vx = 12 • 10-2 м3, V2 = 8 • 10-2 м3, В баллонах находится идеальный газ в количестве v = 3 моля. Первый баллон поддерживается при температуре = 0 °С. До какой температуры нужно нагреть второй баллон, чтобы в нем осталась одна треть общего количества газа? Каким будет давление в сосудах?
9.77.    Определить отклонение маятника от положения равновесия, вызванное тепловым движением шарика маятника. Температура воздуха равна Т, масса шарика — т, длина нити — I.
9.78*. Определить температуру газа, при которой средняя квадратичная скорость молекул водорода больше их наиболее вероятной скорости на Ли = 400 м/с. Найти среднюю арифметическую скорость молекул водорода при этой температуре.

9.81.    При давлении р1 = 780 мм рт. ст. объем воздуха V = 5 л. Найти его объем при давлении р2 = 750 мм рт. ст. Температура воздуха не меняется.
9.82.    Во сколько раз изменится давление газа в цилиндре, если его объем уменьшить, продвинув поршень на — высоты цилиндра? Темпе-
О
ратура газа не меняется.
9.83.    Газ изотермически сжат от объема Vx = 8 л до объема V2 = б л.
Давление при этом возросло на Ар = 4 • 103 Па. Определить первоначальное давление.
9.84.    Идеальный газ сжимают изотермически так, что давление газа изменяется в п = 3 раза. На сколько при этом изменяется объем, занимаемый газом, если начальный объем газа Vt = 6 л?
9.85.    Идеальный газ расширяют изотермически так, что объем газа изменяется в п = 1,4 раза, а давление — на Ар = 2 атм. Найти начальное давление газа.
9.86.    При изотермическом сжатии объем газа уменьшился на AV1 = 1 л. При этом его давление возросло на т)2 = 20% . На сколько процентов увеличилось бы давление, если бы объем был уменьшен на
AV2 = 2 л?
9.87.    Выходное отверстие велосипедного насоса диаметром d — 4 мм зажато пальцем (рис. 9.1). Найти силу давления воздуха на палец в тот момент, когда поршень, сжимая воздух, не доходит до конца насоса на расстояние 12 — 2 см. Длина насоса = 42 см. Процесс считать изотермическим. 

9.88.    Чтобы изотермически уменьшить объем газа в цилиндре с поршнем в п раз, на поршень поместили груз массой т. Какой массы тг груз следует добавить, чтобы объем газа изотермически уменьшился еще в k раз?
9.89.    В цилиндре под поршнем находится воздух.
Поршень имеет форму, показанную на рисунке 9.2. Масса поршня М = 6 кг, площадь сечения цилиндра S0 =
= 20 см. Атмосферное давление pQ = 105 Па. Какой массы груз надо положить на поршень, чтобы объем воздуха уменьшился в два раза? Температура воздуха постоянна, трение не учитывать.
9.90.    При давлении р = 2 * 106 Па идеальный газ занимает объем
V    = 5 л. В результате изотермического расширения его объем увеличился на AV = 1 л, а концентрация молекул стала равной п = 3,62 • 1026 м“3. При какой температуре протекал этот процесс?
9.91.    Электрическая лампа наполнена азотом при давлении р± =
= 600 мм рт. ст. Объем лампы V = 500 см3. Какая масса воды войдет в лампу, если у нее отломить кончик под водой при давлении р2 = 760 мм рт. ст.?
9.92.    Песок насыпают в цилиндр и плотно закрывают поршнем. При суммарном объеме песка и воздуха Vx давление воздуха равно р1( а при суммарном объеме V2 давление воздуха равно р2• Найти объем песка V, если температура неизменна.
9.93.    Упругий шар, наполненный газом и имеющий радиус = 10 см при внешнем давлении pQ = 105 Па, помещен под колокол воздушного насоса. Каким должно стать внешнее давление рх, чтобы радиус шара увеличился на Аг = 0,5 см? Давление, создаваемое оболочкой, р = аг2, где а = 1,66 • 107 Па/м2 иг — радиус шара. Температуру газа считать неизменной.
9.94.    В цилиндре под поршнем изобарически охлаждают газ объемом
VI    = 10 л от температуры Т1 = 323 К до температуры Т2 = 273 К. Каков объем газа при температуре Т21
9.95.    Газ в трубе плавильной печи охлаждается от температуры = 1150 °С до температуры t2 — 200 °С. Во сколько раз увеличивается
плотность газа при этом? Давление газа не меняется.

9.96.    Идеальный газ нагревают от температуры Т± — 300 К до температуры Т2 = 750К при постоянном давлении, в результате чего его
объем увеличивается на AV = 6 • 10_3 м3. Определить первоначальный объем газа.
9.97.    При сгорании топлива в цилиндре дизельного двигателя во время предварительного расширения объем газа увеличился в п = 2,2 раза при постоянном давлении. Определить изменение температуры, если начальная температура газа Тх = 1650 К.
9.98.    Температура воздуха в горизонтальном цилиндре с поршнем ty = 7 °С. На какое расстояние переместится поршень при нагревании воздуха на ДТ = 20 К, если вначале он находился на расстоянии I = 14 см от торца цилиндра?
9.99.    При нагревании некоторой массы идеального газа на один градус при постоянном давлении его объем увеличился на а = часть. Найти первоначальную температуру газа.
9.100.    Газовый термометр состоит из шара с припаянной к нему горизонтальной стеклянной трубкой (рис. 9.3). Капелька ртути, помещенная в трубку, отделяет объем шара от внешнего пространства. Площадь поперечного сечения трубки S = ОД см2. При температуре Т1 = 273 К капелька находилась на расстоянии 1г = 30 см от поверхности шара, при температуре Т2 = 278 К — на расстоянии 12 = 50 см. Найти объем шара и зависимость Т (/). Давление считать постоянным.
9.101.    При температуре = 27 °С давление газа в закрытом сосуде pt = 75 кПа. Каким будет давление при температуре t2 = -13 °С?
9.102.    В нерабочем состоянии при температуре = 7 °С давление газа в колбе газонаполненной электрической лампы накаливания рх = 80 кПа. Найти температуру газа в горящей лампочке, если давление в рабочем режиме возрастает до р2 = 100 кПа.
9.103.    При какой температуре находился газ в закрытом сосуде, если при нагревании его на АТ = 140 К давление возрастает вп = 1,5 раза?
9.104.    На сколько изменилось давление воздуха в автомобильной шине при повышении температуры на АТ — 30 К, если при температуре Tj = 270 К давление было = 3,6 • 105 Па? Изменением объема шины пренебречь. 

9.105.    Сосуд с газом плотно закрыт пробкой, площадь сечения которой S = 2,5 см2. До какой температуры надо нагреть газ, чтобы пробка вылетела из сосуда, если сила трения, удерживающая пробку, F = 12 Н? Начальное давление воздуха в сосуде рх = 105 Па, а начальная температура = -3 °С.
9.106.    Давление воздуха в сосуде р0 = 768 мм рт. ст. Объем цилиндра разрежающего насоса в г| = 3 раза меньше объема сосуда. Какое давление рп установится в сосуде после л = 3 ходов поршня? Изменением температуры пренебречь.
9.107.    Объем сосуда У = 300 см3, объем цилиндра разрежающего насоса Vx = 200 см3. После п = 6 ходов поршня в сосуде установилось давление р = 35 мм рт. ст. Каково было первоначальное давление р0 газа в сосуде? Изменением температуры пренебречь.
9.108*. В баллоне объемом У = 1,5 л находится воздух при нормальном давлении. За сколько ходов поршня насоса, имеющего объем цилиндра Vl = 100 см3, можно понизить давление в баллоне в ц = 100 раз? Температуру считать постоянной.
9.109.    Компрессор, обеспечивающий работу отбойных молотков, за-сасывает из атмосферы У0 = 100 л воздуха в секунду. Сколько отбойных молотков может работать от этого компрессора, если для работы одного молотка необходимо У — 100 см3 воздуха в секунду при давлении р = 5 * 106 Па? Атмосферное давление р0 = 105 Па.
9.110.    Во сколько раз изменится давление в резервуаре пневматического тормоза трамвайного вагона после п = 250 ходов поршня насоса, если объем резервуара У = 30 л? Насос за одно качание подает ДУ = 600 см3 воздуха при нормальном атмосферном давлении. Изменением температуры пренебречь. Начальное давление в резервуаре равно нормальному атмосферному.
9.111.    За сколько ходов поршня насоса с рабочим объемом ДУ можно повысить давление от атмосферного р0 до р в сосуде объемом У? Изменением температуры пренебречь.
9.f ■)
9.117.    Во сколько раз изменится средняя квадратичная скорость мо-лекул идеального газа при увеличении его объема в пх = 2 раза? Давление газа при этом увеличилось в п2 = 3 раза, масса неизменна.
9.118.    В закрытом цилиндрическом сосуде находится газ при нормаль-ных условиях. Сосуд расположен горизонтально и разделен подвижным поршнем в отношении Vx : V2 = 1 : 2. В каком отношении поршень будет делить сосуд, если его меньшую часть нагреть до = 127 °С, а большую охладить до t2 = -123 °С?
9.119.    В закрытом сосуде цилиндрической формы находится газ при температуре — О °С. Внутри сосуд перегорожен легким, не проводящим тепло поршнем радиуса г = 2 см на две части объемами Vx = 10 см3 и V2 — 50 см3. Поршень находится в равновесии. На какое расстояние переместится поршень, если большую часть газа нагреть на 30 К? Температура в другой части не меняется.
9.120°. В цилиндрическом сосуде с газом находится в равновесии тяжелый поршень (рис. 9.4). Масса газа и температура под поршнем и над ним одинаковы. Отношение объема над поршнем к объему под поршнем равно 3. Каким будет это отношение, если температуру в сосуде увеличить в 2 раза?

9.105.    Сосуд с газом плотно закрыт пробкой, площадь сечения которой S = 2,5 см2. До какой температуры надо нагреть газ, чтобы пробка вылетела из сосуда, если сила трения, удерживающая пробку, F = 12 Н? Начальное давление воздуха в сосуде рх = 105 Па, а начальная температура = -3 °С.
9.106.    Давление воздуха в сосуде р0 = 768 мм рт. ст. Объем цилиндра разрежающего насоса в г| = 3 раза меньше объема сосуда. Какое давление рп установится в сосуде после л = 3 ходов поршня? Изменением температуры пренебречь.
9.107.    Объем сосуда У = 300 см3, объем цилиндра разрежающего насоса Vx = 200 см3. После п = 6 ходов поршня в сосуде установилось давление р = 35 мм рт. ст. Каково было первоначальное давление р0 газа в сосуде? Изменением температуры пренебречь.
9.108*. В баллоне объемом У = 1,5 л находится воздух при нормальном давлении. За сколько ходов поршня насоса, имеющего объем цилиндра Vl = 100 см3, можно понизить давление в баллоне в ц = 100 раз? Температуру считать постоянной.
9.109.    Компрессор, обеспечивающий работу отбойных молотков, за-сасывает из атмосферы У0 = 100 л воздуха в секунду. Сколько отбойных молотков может работать от этого компрессора, если для работы одного молотка необходимо У — 100 см3 воздуха в секунду при давлении р = 5 * 106 Па? Атмосферное давление р0 = 105 Па.
9.110.    Во сколько раз изменится давление в резервуаре пневматического тормоза трамвайного вагона после п = 250 ходов поршня насоса, если объем резервуара У = 30 л? Насос за одно качание подает ДУ = 600 см3 воздуха при нормальном атмосферном давлении. Изменением температуры пренебречь. Начальное давление в резервуаре равно нормальному атмосферному.
9.111.    За сколько ходов поршня насоса с рабочим объемом ДУ можно повысить давление от атмосферного р0 до р в сосуде объемом У? Изменением температуры пренебречь.
9.112.    Компрессор захватывает при каждом ходе поршня ДУ = 4 л воздуха при нормальном атмосферном давлении и температуре fj = -3 °С и нагнетает его в резервуар емкостью У = 1,5 м3.f ■)
9.117.    Во сколько раз изменится средняя квадратичная скорость мо-лекул идеального газа при увеличении его объема в пх = 2 раза? Давление газа при этом увеличилось в п2 = 3 раза, масса неизменна.
9.118.    В закрытом цилиндрическом сосуде находится газ при нормаль-ных условиях. Сосуд расположен горизонтально и разделен подвижным поршнем в отношении Vx : V2 = 1 : 2. В каком отношении поршень будет делить сосуд, если его меньшую часть нагреть до = 127 °С, а большую охладить до t2 = -123 °С?
9.119.    В закрытом сосуде цилиндрической формы находится газ при температуре — О °С. Внутри сосуд перегорожен легким, не проводящим тепло поршнем радиуса г = 2 см на две части объемами Vx = 10 см3 и V2 — 50 см3. Поршень находится в равновесии. На какое расстояние переместится поршень, если большую часть газа нагреть на 30 К? Температура в другой части не меняется.
9.120°. В цилиндрическом сосуде с газом находится в равновесии тяжелый поршень (рис. 9.4). Масса газа и температура под поршнем и над ним одинаковы. Отношение объема над поршнем к объему под поршнем равно 3. Каким будет это отношение, если температуру в сосуде увеличить в 2 раза?

9.138.    Определить давление кислорода в баллоне объемом V = 1 м3 при температуре t — 27 °С. Масса кислорода m — 1 кг.
9.139.    Какой объем занимают два моля идеального газа в фотосфере Солнца? Температура фотосферы Т = 6000 К, давлениер = 1,25 • 102 Па.
9.140.    Каким может быть наименьший объем баллона, содержащего кислород массой m = 6,4 кг, если его стенки при температуре t = 20 °С выдерживают давление р = 1568 Н/см2?
9.141.    Доказать, что один моль идеального газа при нормальных ус-ловиях занимает объем V = 22,4 л. Сколько молекул при этом находится в единице объема (закон Авогадро)?

9.142.    Баллон емкостью V = 20 л содержит углекислый газ массой т — 0,5 кг при давлении р = 1,3 МПа. Определить температуру газа.
9.143.    Баллон емкостью V = 12 л содержит углекислый газ. Давление газа р = 1 МПа, температура Т = 300 К. Определить массу газа.
9.144.    При температуре Т = 309 К и давлении р = 0,7 МПа плотность газа р = 12 кг/м3. Определить молярную массу газа.
9.145.    Высота пика Ленина на Памире 7134 м. Атмосферное давление на этой высоте 3,8 * 104 Па. Определить плотность воздуха на вершине пика при температуре 0 °С. Чему равно число молекул воздуха в единице объема при данных условиях?
9.146.    Используя уравнение состояния идеального газа, доказать, что плотность любого газа равна половине плотности водорода рН2 (взятого
при тех же условиях), умноженной на относительную молекулярную массу этого газа Мг, т. е. р = рщМг.
9.147.    До какой температуры Тх при постоянном давлении р = 105 Па надо нагреть кислород, чтобы его плотность стала равна плотности водорода при том же давлении и температуре Т2 = 200 К?
9.148.    Найти формулу некоторого соединения углерода с водородом, если известно, что это вещество массой m = 0,66 г в газообразном состоянии при температуре t = 27 °С в объеме V = 1 дм3 создает давление р = 105 Па.
9.149.    Найти формулу соединения углерода с кислородом, если известно, что это вещество в газообразном состоянии массой m = 1 г при температуре t = 27 °С и давлениир = 5,6 • 104 Па занимает объем V — 1 дм3.
9.150.    При температуре t = 27 °С и давлении р = 4,155 * 105 Па плотность газа р = 2,833 кг/м3. Известно, что молекулы этого газа представляют собой соединение атомов азота N1/ и водорода Н\. Определить молекулярную формулу этого соединения.
9.151.    Пар органического соединения углерода Cg2, водорода Н{ и кис-лорода О*6, формула которого (C3HgO)n, массой пг = 716 мг занимает при
температуре t = 200 °С и давлении р = 105 Па объем V = 242,6 см3. Найти число п.
9.152.    Вода полностью заполняет сосуд емкостью V = 1 л. Температура воды f = 27 °С. Оценить давление, которое могло бы установиться внутри сосуда, если бы исчезли силы взаимодействия между молекулами воды. 

9.153.    В сосуде под поршнем находится т1 = 1 г азота. Площадь
поршня S = 10 см2, его масса т= 1 кг. Азот нагревают на АТ — 10 К. На какую высоту при этом поднимется поршень? Давление над поршнем р0 = 105 Па. Трения в системе нет.
9.154.    Один моль гелия находится при температуре Т — 300 К в вер-тикальном закрытом теплоизолированном цилиндре с поршнем массой
= 2 кг и диаметром d = 10 см. На поршень ставят гирю массой т2 = 3 кг. При этом поршень опускается на Л = 5 см. Определить установившуюся температуру газа, если атмосферное давление р0 = 105 Па.
9.155.    Один моль идеального газа расширяется изобарически. При этом оказалось, что а = V/T = 2,8 • 10~3 м3/К. Определить концентрацию молекул газа при температуре Тх = 103 К.
9.156°. Состояние идеального газа массой m изменяется в соответст-
2
вии с законом = а, где а — известная константа. Определить зависимость давления газа от его объема в этом процессе. Молярная масса газа равна М.
9.157°. Процесс в идеальном газе идет так, что давление и объем связаны равенством pjv — В. Когда температура газа достигает значения Г, процесс продолжается при другом характере зависимости давления от
объема: р =. = 15 °С и давлении р = 105 Па. При нагревании баллона до температуры t2 = 37 °С через клапан выходит водород массой m — 6 кг, вследствие чего давление не изменяется. Определить объем баллона.
9.160.    При аэродинамическом торможении в атмосфере планеты температура внутри автоматического спускаемого аппарата увеличилась от температуры = 20 °С до температуры t2 = 80 °С. Какую часть воздуха необходимо выпустить, чтобы давление внутри аппарата не изменилось? 

9.161.    В неплотно закрытом баллоне объемом V = 10~2 м3 при темпе-ратуре Т = 293 К и давлении р = 10 МПа находился водород. Сколько водорода было потеряно, если из оставшегося водорода может образоваться вода массой m — 0,5 кг?
9.162.    Стеклянная колба закрыта пробкой и взвешена при температуре tj = 15 °С. Открыв пробку, колбу нагрели до температуры t2 = 80 °С. При следующем взвешивании масса колбы оказалась на m = 0,25 г меньше. Чему равен объем колбы?
9.163.    В баллоне, объем которого V = 10 л, находится гелий под дав-лением pl = 105 Па при температуре tx = 27 °С. После того как из баллона был взят гелий массой m — 10 г, давление в баллоне понизилось до р2 — 0,9 • 105 Па. Определить температуру гелия, оставшегося в баллоне.
9.164.    В баллоне емкостью V = 12 л находится азот массой тг = 1,5 кг при температуре t1 = 37 °С. Каким станет давление в баллоне при температуре t2 = 50 °С, если выпустить г\ = 35% азота? Найти начальное давление азота.
9.165.    В сосуде объемом V = 1 л находится идеальный газ. Сколько молекул газа нужно выпустить из сосуда, чтобы при понижении температуры в k2 — 2 раза его давление уменьшилось в = 4 раза? Первоначальная концентрация молекул газа пх = 2 * 1026 м-3.
9.166.    Когда из сосуда выпустили некоторое количество газа, давление в нем упало на = 40%, а абсолютная температура на г\2 = 20%. Какую часть газа выпустили?
9.167.    Определить, во сколько раз изменится концентрация молекул газа, если изобарически уменьшить абсолютную температуру в а = 7 раз, а затем количество газа уменьшить в Ъ = 14 раз при том же давлении. часть газа и увеличить температуру в 6 раз, поддерживая давление постоянным?
9.171.    По трубе, площадь сечения которой S = 5 ■ 10 4 м2, течет угле-кислый газ (С02) под давлением р = 3,92 * 105 Па при температуре Т = 280 К. Найти среднюю скорость протекания газа по трубе, если через поперечное сечение за время / = 10 мин проходит газ массой m = 20 кг.
9.172.    В ампуле объемом V = 3 см3, из которой откачан воздух, содержится радий массой m = 5 • Ю-10 г в течение времени t = 1 год. Известно, что из радия массой m0 = 1 г за время t0 = 1 с вылетает п0 = 3,7 • 1010 ядер гелия. Найти давление образовавшегося гелия при температуре Т ~ 300 К. Считать, что скорость вылета числа частиц за время наблюдения остается неизменной; объем, занимаемый в ампуле радием, пренебрежимо мал.
9.173.    Аэростат объемом V — 300 м3 наполняется молекулярным во-дородом при температуре Т = 300 К и давлении р = 105 Па. Какое время будет производиться наполнение оболочки аэростата, если из баллонов каждую секунду переходит в аэростат Ат = 25 г водорода? До наполнения газом оболочка аэростата водорода не содержала; газ считать идеальным.
9.174.    Закрытый сосуд объемом V = 10 см3 имеет трещину, через которую ежесекундно проникает AN = 106 частиц газа. Какое время понадобится для наполнения сосуда до нормального давления р = 105 Па, если скорость проникновения частиц остается постоянной и начальное давление в сосуде р0 — 0? Температура сосуда с газом Т = 273 К.
9.175.    Для дальней космической связи используется спутник объемом V = 1000 м3, наполненный воздухом, находящимся при нормальных условиях. Метеорит пробивает в корпусе спутника отверстие площадью S = 1 см2. Оценить время, через которое давление внутри спутника изменится на г| = 1%. Температуру газа считать неизменной.
9.176.    Цилиндрический сосуд длиной I = 85 см
разделен на две части легкоподвижным поршнем (рис. 9.20). При каком положении поршня давление в обеих частях цилиндра будет одинаково, если одна часть заполнена кислородом, а другая водородом такой же массы? Температура в обеих частях цилиндра одинакова.Рис. 9.20

9.177.    Цилиндрический сосуд длиной L = 85 см разделен на две части легкоподвижным поршнем.
В одной части сосуда находится водород, в другой — кислород той же массы. При каком отношении температур поршень будет делить сосуд на две равные части? ,
9.178°. В вертикально расположенном цилиндре находится газ массой т = 0,01 кг. Он отделен от атмосферы поршнем, соединенным с дном пружиной жесткостью k = 20 Н/м (рис. 9.21). При температуре Тг = 290 К поршень расположен на расстоянии h = 0,2 м от дна цилиндра. До какой температуры Т2 надо нагреть газ, чтобы поршень поднялся до высоты Н = 0,5 м? Молярная масса р = 29 кг/моль.
9.179*. Гелий массой m = 20 г бесконечно мед- Р ленно переводят из состояния, в котором газ зани- Рг мает объем Vx = 32 л при давлении рх = 4,1 • 105 Па, в состояние с термодинамическими параметрами р V2 = 9 л и р2 = 15,5 • 105 Па (рис. 9.22). До какой на- ибольшей температуры нагревается газ в этом процессе?
9.180*. Два одинаковых сосуда соединены трубкой с клапаном, про-пускающим газ из одного баллона в другой при разности давлений Ар > 1,1 атм. Сначала в одном баллоне был вакуум, а в другом идеальный газ при температуре гх = 19 °С под давлением рх = 1 атм. Затем оба баллона нагрели до температуры t2 — 107 °С. Найти давление газа в баллоне, где был вакуум.

9.181.    В колбе емкостью V = 4 л находятся кислород и азот при тем-пературе 0 °С. Определить давление на стенки сосуда, если массы газов Щ = тг = 1г.
9.182.    Какое давление воздуха должно быть в сосуде, объем которого Vl = 10 л, чтобы при соединении его с сосудом объемом V2 = 30 л, в котором находится воздух при давлении р2 = 105 Па, установилось давление р = 3 • 105 Па? Температуру считать постоянной.

9.184.    Два сосуда, заполненные разными идеальными газами, соединены трубкой с краном. Давление в сосудах р1 и р2, а число молекул Nj и N2 соответственно. Каким будет давление в сосудах, если открыть кран соединительной трубки? Температуру считать постоянной.
9.185.    В сосуде объемом Vx находится одноатомный газ при давлении рх и температуре Tv а в сосуде объемом V2 такой же газ при давлении р2 и температуре Т2. Какое давление и температура установятся в сосудах при их соединении? Теплообменом с окружающей средой и стенками сосудов пренебречь.
9.186.    Приближенно воздух можно считать смесью азота (гц = 80% по массе), кислорода (ri2 = 16%) и углекислого газа (г|3 = 4%). Найти эффективную молярную массу М воздуха, т. е. молярную массу такого газа, который при одинаковых параметрах со смесью будет иметь ту же массу.
9.187.    В сосуде при давлении р = 105Па и температуре t = 27 °С находится смесь азота, кислорода и гелия, массы которых равны. Найти плотность смеси газов.
9.188.    За один вдох в легкие человека попадает воздух объемом V = 0,5 л. Сколько молекул кислорода содержится в таком объеме воздуха, если доля кислорода в нем составляет г| = 20% ?
9.189.    Плотность газа, состоящего из смеси гелия и аргона, р = = 2 кг/м3 при давлении р = 150 кПа и температуре t — 27 °С. Сколько атомов гелия содержится в газовой смеси объемом V = 1 см3?
9.190.    Сосуд разделен пополам полупроницаемой перегородкой. Объем каждой части V = 1 л. В левую половину введены водород массой тх = 2 г и азот массой т2 = 28 г. Справа от перегородки — вакуум. Какие давления установятся в обеих частях сосуда, если перегородка пропускает только водород, а температура остается постоянной Т = 373 К?
9.191.    Закрытый сосуд разделен на две равные половины поршнем, который может перемещаться без трения. Давление и температура в обеих половинах одинаковы. В левой части находится чистый газ 1, в правой — смесь газов 1 и 2, причем их парциальные давления равны. В некоторый момент поршень становится проницаемым для газа 2. Во сколько раз г\ увеличится объем левой части после того, как установится равновесие?

9.192.    На гладком столе лежит цилиндрический невесомый сосуд длиной U разделенный герметичной перегородкой на две равные части, в одной из которых находится под некоторым давлением азот, а в другой — углекислый газ под давлением, вдвое большим. Температура газов одинакова. В некоторый момент перегородка теряет герметичность. На сколько и в каком направлении окажется смещенным сосуд после того, как газы перемешаются?

9.194.    Некоторое количество водорода находится при температуре Тг — 200 К и давлении рг = 400 Па. Газ нагревают до температуры Т2 = 104К, при которой молекулы водорода полностью распадаются на атомы. Определить давление газа, если его объем и масса не изменились.
9.195.    В сосуде находится озон (03) при температуре — 527 °С. Через некоторое время он полностью превращается в кислород (02), а температура устанавливается t2 = 127 °С. На сколько процентов изменилось давление в баллоне?
9.196.    При комнатной температуре четырехокись азота частично дис-социирует, превращаясь в двуокись: N204 *=► 2N02. В сосуд, из которого откачан воздух, объемом V — 250 см3 вводится жидкость N204 массой m — 0,92 г при температуре tl = 0 °С. При температуре t2= 27 °С жидкость целиком испаряется, и давление становится равным р = 128 кПа. Определить долю т| четырехокиси азота, которая диссоциировала.
9.197.    В колбе находится двухатомный газ. Под действием ультра-фиолетового излучения распалось на атомы а = 12% молекул. При этом в колбе установилось давление р = 93 кПа. Найти давление газа в недис- социированном состоянии.
9.198.    При температуре t = 1000 °С распадается на атомы т\ = 11,6% молекул иода 12. Какова масса паров иода, находящихся в сосуде объемом V = 0,5 л, если давление в нем при данной температуре р = 93 кПа? 

9.199.    В закрытом сосуде находится идеальный двухатомный газ. При увеличении температуры в п = 3 раза давление газа увеличилось в k = 3,15 раза. Сколько процентов молекул от их начального количества распалось на атомы?
9.200.    В сосуде находится смесь азота и водорода. При температуре Т, когда азот полностью диссоциировал на атомы, а диссоциацией водорода еще можно пренебречь, давление в сосуде равно р. При температуре 27\ когда оба газа полностью диссоциировали, давление в сосуде равно Зр. Каково отношение числа атомов азота и водорода в смеси?
9.201.    Кислород массой m = 8 г при температуре £0 = 27 °С занимает объем V — 200 л. Определить давление в этом объеме, если газ превращен в полностью ионизированную плазму при температуре Т = 106К, сравнить его с первоначальным давлением.

9.202.    При какой температуре давление воздуха на дно сосуда высотой Н = 1,2 м будет в k = 1,00005 раза больше давления на крышку сосуда?
9.203.    Со дна реки поднимается воздушный пузырек. У поверхности его объем увеличивается в г\ = 1,5 раза. Найти глубину реки, считая, что температура воды всюду одинакова. Атмосферное давление р0 = 105Па.
9.204.    Тонкостенный резиновый шар радиусом = 3 см наполнен воздухом при температуре tx = 27 °С и нормальном атмосферном давлении. Каким станет радиус шара, если его опустить в воду при температуре t2 = 7 аС на глубину Л = 30 м? Силы упругости, возникающие в резине, не учитывать.
9.205.    На дне сосуда, заполненного воздухом, лежит полый стальной шарик радиусом г = 2 см и массой m = 5 г. До какого давления нужно сжать воздух в сосуде, чтобы шарик поднялся вверх? Температура постоянна, t = 20 °С.
9.206.    Тонкостенный резиновый шар массой т± = 50 г наполнен азотом и погружен в озеро на глубину h = 100 м. Найти массу азота, если шар находится на этой глубине в положении равновесия. Атмосферное давление р0 = 760 мм рт. ст. Температура на глубине озера t — 4 °С. Силы упругости, возникающие в резине, не учитывать.

9.207.    Полый стальной куб со стороной а — 1 м заполнен воздухом при нормальных условиях и запаян. Куб плавает в воде так, что его нижняя грань расположена горизонтально. Толщина стенки куба b = 4 мм. Вычислить давление воды на нижнюю грань куба.
9.208.    Атмосфера Венеры почти полностью состоит из углекислого газа. Температура у поверхности планеты около t = 500 °С, а давление около р = 100 атм. Какой объем должен иметь исследовательский зонд массой т = 1 т, чтобы плавать в нижних слоях атмосферы Венеры?
9.209*. Шар сделан из прочной нерастяжимой воздухонепроницаемой оболочки. Объем оболочки V = 83,1 м3, масса шара т = 5 кг. На какую высоту сможет подняться шар? Давление у поверхности Земли р0 =
— 105 Па. Оно уменьшается вдвое при подъеме на каждые й0 = 5 км высоты. Температуру воздуха в верхних слоях атмосферы принять t — 73 °С.
9.210.    Полый шар с жесткой оболочкой, масса которой т — 10 г, наполнен водородом. Объем водорода V = 10 л. Температура водорода и окружающего шар воздуха t = 0 °С. Найти давление водорода в шаре, если подъемная сила шара равна нулю. Атмосферное давление р0 = 105 Па.
9.211.    В воздушном шаре весь наполняющий его водород заменили гелием. Во сколько раз изменилась подъемная сила шара? Весом оболочки шара пренебречь.
9.212.    Воздушный шар объемом V заполнен подогретым воздухом (молярная масса воздуха М). Температура окружающего воздуха Г, давление р. Температура внутри шара Tv Найти подъемную силу. (Излишек воздуха может свободно выходить из отверстия внизу шара, через которое осуществляется подогрев, массой оболочки шара пренебречь.)
9.213.    При каком наименьшем радиусе станет подниматься воздушный шар, наполненный гелием, если поверхностная плотность материала обо-лочки а = 50 г/м2, давление воздуха р = 105 Па, а температура t = 27 °С?
9.214.    Оболочка стратостата открыта снизу и заполнена частично водо-родом и частично воздухом. Масса водорода т. Какова подъемная сила стратостата? Массой оболочки пренебречь.
9.215.    Пробирка, расположенная горизонтально, заполнена ртутью так, что между дном пробирки и ртутью имеется пузырек воздуха. Когда пробирка ставится вертикально открытым концом вверх, объем пузырька уменьшается втрое. Чему равно атмосферное давление, если известно, что диаметр пробирки d = 1 мм и содержит она т = 16 г ртути?

9.216.    Посередине запаянной с обоих концов горизонтальной трубки находится столбик ртути длиной h = 10 см. В обеих половинах трубки находится воздух под давлением р0 = 760 мм рт. ст. Длина трубки I = 1 м. На какое расстояние сместится столбик ртути, если трубку поставить вер-тикально?
9.217.    В стеклянной трубке находится воздух, закрытый столбиком ртути длиной 1Х = 8 см. Если держать трубку открытым концом вверх, то длина воздушного столбика 12 = 4 см. Если держать трубку открытым концом вниз, то длина воздушного столбика h — 5 см. = 8 см. Трубку закрывают и поднимают еще на расстояние 12 = 44 см. Какую часть трубки при этом занимает воздух? Атмосферное давление р0 = 760 мм рт. ст.
9.220.    Узкую цилиндрическую трубку, запаянную с одного конца, длиной I = 45 см погружают открытым концом в сосуд с ртутью на глубину Н = 40 см. Атмосферное давление р0 = 76 см рт. ст. Какова будет высота столбика ртути в трубке?
9.221.    Вертикальная барометрическая трубка опущена в широкий сосуд с ртутью. Столб ртути в трубке над поверхностью ртути имеет высоту Л0 = 40 мм, а столб воздуха над ртутью hx = 190 мм. На какое расстояние надо опустить трубку, чтобы уровень ртути в трубке сравнялся с уровнем ртути в сосуде? Атмосферное давление р0 = 760 мм рт. ст.
9.222.    Трубку длиной b = 76 см, запаянную с одного конца, погружают в вертикальном положении открытым концом в сосуд с ртутью. На каком расстоянии от поверхности ртути должен находится запаянный конец трубки, чтобы уровень ртути в ней был ниже уровня в сосуде на расстояние h = 7,6 см? Атмосферное давлениер0 = 105 Па.
9.223.    В пространство над ртутным столбиком в трубке барометра попало немного воздуха. Когда барометр показывал давление рх = = 748 мм рт. ст., длина части трубки, занятой воздухом, была I = 60 мм. Трубку выдвинули из сосуда с ртутью еще на расстояние Лh = 21 мм, после чего барометр стал показывать давление р2 — 751 мм рт. ст. Считая температуру постоянной, определить атмосферное давление р0. 

9.224.    В мензурке высотой Л = 0,4 м и сечением 5=12 см2, закрытой тонким невесомым поршнем, находится газ, молярная масса которого М = 0,029 кг/моль. Поршень опускают и освободившуюся часть мензурки до краев заливают ртутью. При каких значениях температуры газа можно найти такое положение поршня, при котором поршень будет находиться в равновесии (т. е. ртуть, налитая в мензурку, не будет выбрасываться давлением газа)? Масса газа в мензурке = 0,07 г, внешним атмосферным давлением пренебречь.
9.225.    Стеклянная трубка, запаянная с одного конца, расположена горизонтально. В трубке находится воздух, отделенный от атмосферы
столбиком ртути длиной I. ,
Li
атмосферное давление р0. На какое расстояние А сместится ртуть в трубке, если ее вращать вокруг вертикальной оси, проходящей через открытый
конец, с угловой скоростью со = (g — ускорение свободного падения).
9.226.    Опрокинутый стакан высотой Н = 10 см погружается в воду так, что дно стакана оказывается вровень с уровнем жидкости (рис. 9.23). На каком уровне А установится жидкость в стакане, если давление воздуха на поверхность воды р0 — 0,1 атм?
в воде так, что его края находятся на уровне ее поверхности (рис. 9.24, а). Этот же стакан с воздухом, нагретым до температуры tx = 87 °С, погружают в воду вверх дном (рис. 9,24, б), при этом стакан и воздух в нем охлаждаются до температуры воды, равной t2 = 27 °С. На какую глубину Н надо погрузить стакан, чтобы он не всплывал и не тонул? Атмосферное давление р0 = 0,1 МПа. Давлением насыщенных паров в стакане пренебречь.
9.228.    К стеклянному баллону объемом V = 25 л припаяна трубка длиной I = 1 м и сечением 5 = 2 см2 (рис. 9.25). Своим концом стеклянная 

 

 

Шкала температуры. Шкала Цельсия, Фаренгейта, Кельвина, Реомюра

История

Слово «температура» возникло в те времена, когда люди считали, что в более нагретых телах содержится большее количество особого вещества — теплорода, чем в менее нагретых. Поэтому температура воспринималась как крепость смеси вещества тела и теплорода. По этой причине единицы измерения крепости спиртных напитков и температуры называются одинаково — градусами.

Из того, что температура — это кинетическая энергия молекул, ясно, что наиболее естественно измерять её в энергетических единицах (т.е. в системе СИ в джоулях). Однако измерение температуры началось задолго до создания молекулярно-кинетической теории, поэтому практические шкалы измеряют температуру в условных единицах — градусах.

Шкала Кельвина

В термодинамике используется шкала Кельвина, в которой температура отсчитывается от абсолютного нуля (состояние, соответствующее минимальной теоретически возможной внутренней энергии тела), а один кельвин равен 1/273.16 расстояния от абсолютного нуля до тройной точки воды (состояния, при котором лёд, вода и водяной пар находятся в равновесии). Для пересчета кельвинов в энергетические единицы служит постоянная Больцмана. Используются также производные единицы: килокельвин, мегакельвин, милликельвин и т.д.

Шкала Цельсия

В быту используется шкала Цельсия, в которой за 0 принимают точку замерзания воды, а за 100° точку кипения воды при атмосферном давлении. Поскольку температура замерзания и кипения воды недостаточно хорошо определена, в настоящее время шкалу Цельсия определяют через шкалу Кельвина: градус Цельсия равен кельвину, абсолютный ноль принимается за −273,15 °C. Шкала Цельсия практически очень удобна, поскольку вода очень распространена на нашей планете и на ней основана наша жизнь. Ноль Цельсия — особая точка для метеорологии, поскольку замерзание атмосферной воды существенно всё меняет.

Шкала Фаренгейта

В Англии и, в особенности, в США используется шкала Фаренгейта. В этой шкале на 100 градусов раздёлен интервал от температуры самой холодной зимы в городе, где жил Фаренгейт, до температуры человеческого тела. Ноль градусов Цельсия — это 32 градуса Фаренгейта, а градус Фаренгейта равен 5/9 градуса Цельсия.

В настоящее время принято следующее определение шкалы Фаренгейта: это температурная шкала, 1 градус которой (1 °F) равен 1/180 разности температур кипения воды и таяния льда при атмосферном давлении, а точка таяния льда имеет температуру +32 °F. Температура по шкале Фаренгейта связана с температурой по шкале Цельсия (t °С) соотношением t °С = 5/9 (t °F — 32), то есть изменение температуры на 1 °F соответствует изменению на 5/9 °С. Предложена Г. Фаренгейтом в 1724.

Шкала Реомюра

Предложенна в 1730 году Р. А. Реомюром, который описал изобретённый им спиртовой термометр.

Единица — градус Реомюра (°R), 1 °R равен 1/80 части температурного интервала между опорными точками — температурой таяния льда (0 °R) и кипения воды (80 °R)

1 °R = 1,25 °C.

В настоящее время шкала вышла из употребления, дольше всего она сохранялась во Франции, на родине автора.

 

Пересчёт температуры между основными шкалами

 

Кельвин

Цельсий

Фаренгейт

Кельвин (K)

= K

= С + 273,15

= (F + 459,67) / 1,8

Цельсий (°C)

= K − 273,15

= C

= (F − 32) / 1,8

Фаренгейт (°F)

= K · 1,8 − 459,67

= C · 1,8 + 32

= F

 Сравнение температурных шкал

Описание

Кельвин Цельсий

Фаренгейт

Ньютон Реомюр

Абсолютный ноль

0

−273.15

−459.67

−90.14

−218.52

Температура таяния смеси Фаренгейта (соли и льда в равных количествах)

255.37

−17.78

0

−5.87

−14.22

Температура замерзания воды (нормальные условия)

273.15

0

32

0

0

Средняя температура человеческого тела¹

310.0

36.8

98.2

12.21

29.6

Температура кипения воды (нормальные условия)

373.15

100

212

33

80

Температура поверхности Солнца

5800

5526

9980

1823

4421

¹ Нормальная температура человеческого тела — 36.6 °C ±0.7 °C, или 98.2 °F ±1.3 °F. Приводимое обычно значение 98.6 °F — это точное преобразование в шкалу Фаренгейта принятого в Германии в XIX веке значения 37 °C. Поскольку это значение не входит в диапазон нормальной температуры по современным представлениям, можно говорить, что оно содержит избыточную (неверную) точность. Некоторые значения в этой таблице были округлены.

Сопоставление шкал Фаренгейта и Цельсия

(oF — шкала Фаренгейта, oC — шкала Цельсия)

 

oF

oC

 

oF

oC

 

oF

oC

 

oF

oC

-459.67
-450
-400
-350
-300
-250
-200
-190
-180
-170
-160
-150
-140
-130
-120
-110
-100
-95
-90
-85
-80
-75
-70
-65

-273.15
-267.8
-240.0
-212.2
-184.4
-156.7
-128.9
-123.3
-117.8
-112.2
-106.7
-101.1
-95.6
-90.0
-84.4
-78.9
-73.3
-70.6
-67.8
-65.0
-62.2
-59.4
-56.7
-53.9

 

-60
-55
-50
-45
-40
-35
-30
-25
-20
-19
-18
-17
-16
-15
-14
-13
-12
-11
-10
-9
-8
-7
-6
-5

-51.1
-48.3
-45.6
-42.8
-40.0
-37.2
-34.4
-31.7
-28.9
-28.3
-27.8
-27.2
-26.7
-26.1
-25.6
-25.0
-24.4
-23.9
-23.3
-22.8
-22.2
-21.7
-21.1
-20.6

 

-4
-3
-2
-1
0
1
2
3
4
5
6
7
8
9
10
11
12
13
14
15
16
17
18
19

-20.0
-19.4
-18.9
-18.3
-17.8
-17.2
-16.7
-16.1
-15.6
-15.0
-14.4
-13.9
-13.3
-12.8
-12.2
-11.7
-11.1
-10.6
-10.0
-9.4
-8.9
-8.3
-7.8
-7.2

 

20
21
22
23
24
25
30
35
40
45
50
55
60
65
70
75
80
85
90
95
100
125
150
200

-6.7
-6.1
-5.6
-5.0
-4.4
-3.9
-1.1
1.7
4.4
7.2
10.0
12.8
15.6
18.3
21.1
23.9
26.7
29.4
32.2
35.0
37.8
51.7
65.6
93.3

Для перевода градусов цельсия в кельвины необходимо пользоваться формулой T=t+T0 где T- температура в кельвинах, t- температура в градусах цельсия, T0=273.15 кельвина. По размеру градус Цельсия равен Кельвину.

 

Уравнение идеального газа: макропараметры, температура, давление, объем. Молярная масса газа. Тест

Вопрос 1. Сколько молей газа содержится в объеме 8,3 м3 при давлении 500 Па и температуре 250 К?

Вопрос 2. Найдите объем водорода массой 1 кг при температуре 300 К и давлении 100 кПа.

Вопрос 3. Определите давление (кПа), при котором в объеме 1 м3 идеального газа при температуре 300 К содержится молекул.

Вопрос 4. После того как в комнате протопили печь, температура поднялась с 150 С до 270 С. На сколько процентов уменьшилось число молекул в этой комнате? Давление постоянно.

Вопрос 5. Баллон, содержащий газ под давлением 2,8 МПа, находится на складе при температуре 70С. После того как половина газа была израсходована, баллон внесли в помещение. Какова была температура (К) в помещении, если давление газа в баллоне через некоторое время стало равным 1,5 МПа?

Вопрос 6. Воздушный пузырек поднимается со дна водоема, где температура 279 К. Объем воздуха в пузырьке у поверхности воды в 4 раза больше, чем на дне. Атмосферное давление 100 кПа, температура у поверхности 289 К. Определите глубину водоема.

Вопрос 7. В стеклянной трубке, запаянной с одного конца, расположен столб воздуха, запертый столбиком ртути. Длина столба воздуха в трубке при 294 К составляет 0,12 м. При какой температуре (К) длина столба воздуха увеличивается на 0,06 м?

Вопрос 8. В чашку с ртутью опускают открытую стеклянную трубку, оставляя над поверхностью конец длиной 60 см. Затем трубку закрывают и погружают еще на 30 см. Определите высоту (см) столба воздуха в трубке. Атмосферное давление равно 760 мм.рт.ст.

Вопрос 9. Баллон содержит газ при температуре 170C, сжатый до давления 2 МПа. Каким станет давление (кПа) в баллоне, если выпустить из баллона 80 % газа и нагреть его до 31,50C?

Вопрос 10. На диаграмме VT изображена зависимость объема от температуры при нагревании трех газов — водорода, азота, и углекислого газа. Массы газов одинаковы, все три газа находятся под одним и тем же давлением. Молярные массы: водорода — 2 г/моль, азота — 28 г/моль, углекислого газа — 44 г/моль. Какой график соответствует водороду?

Вопрос 11. В открытом сосуде находится воздух при температуре 283 К. До какой температуры (в 0C) нужно нагреть воздух в сосуде, чтобы плотность уменьшилась в 1,5 раза?

Вопрос 12. В сосуде объемом 1,5 л находится смесь кислорода и углекислого газа. Масса смеси 40 г, температура 300 К, давление 2 МПа. Определите разность масс (г) газов.

Вопрос 13. Три баллона объемами 3; 7 и 5 л наполнены соответственно кислородом при давлении 0,2 МПа, азотом – при 0,3 МПа и углекислым газом – при 0,06 МПа при одной и той же температуре. Баллоны соединяют между собой, при этом образуется смесь той же температуры. Определите давление (кПа) смеси.

Вопрос 14. Баллон с азотом нагревают от 7 до 14970С. Третья часть молекул азота (азот – двухатомный газ) при этом распадается на атомы. Во сколько раз возросло давление газа в баллоне в результате такого процесса?

Вопрос 15. В процессе перехода идеального газа из состояния 1 в состояние 2 давление и объем газа изменяются по закону . Объем газа уменьшили в 4 раза. Во сколько раз изменилась температура?

Вопрос 16. Стеклянный баллон объемом 1 л был наполнен газом до давления 105 Па и взвешен. Его вес оказался равным 10,9898 Н. Затем часть газа была удалена так, что давление в баллоне упало до . Новый вес баллона оказался равным 10,98 Н. Какова плотность газа при атмосферном давлении? Температура постоянна.

Вопрос 17. Закрытый горизонтальный цилиндр разделен на две равные части теплонепроницаемым поршнем. В левой части находится газ при температуре -30С, а в правой при температуре +170С. Если левую часть охладить до -130С, а правую нагреть до +270С, то на какую часть длины цилиндра (в процентах) сместится поршень?

Вопрос 18. В закрытом горизонтальном цилиндрическом сосуде постоянного сечения находится газ под давлением 200 кПа. Сосуд разделен подвижным поршнем в отношении 2:3. Температура в обеих частях сосуда 300 К. В каком отношении поршень будет делить сосуд, если его меньшую часть нагреть на 50 К, а большую охладить на 67 К?

Вопрос 19. Воздушный шар имеет форму сферы и в нижней части через отверстие сообщается с атмосферой. Диаметр шара 10 м, масса оболочки 10 кг. Температура воздуха 27 0С, атмосферное давление 735 мм.рт.ст. До какой температуры (в 0C) нужно нагреть воздух в шаре, чтобы шар взлетел?

Вопрос 20. Баллон, содержащий 1 кг азота при испытании взорвался при температуре 630 К. Какое количество водорода (г) можно хранить в таком баллоне при температуре 270 К, имея десятикратный запас прочности?

I.3. Задачи к теме «Основы молекулярно-кинетической теории» — Мегаобучалка

1. Сколько атомов содержится в 1 г водорода?

2. Сколько атомов содержится в 1 г кислорода?

3. Сколько атомов содержится в 1 г азота?

4. Сколько атомов содержится в 1 г углекислого газа?

5. Сколько атомов содержится в 1 г гелия?

6. Сколько атомов содержится в 1 г воды?

7. Сколько молекул содержится в 1 г водорода?

8. Сколько молекул содержится в 1 г кислорода?

9. Сколько молекул содержится в 1 г азота?

10. Сколько молекул содержится в 1 г углекислого газа?

11. Сколько молекул содержится в 1 г гелия?

12. Сколько молекул содержится в 1 г воды?

13. Каков объем 1 г водорода при нормальных условиях

(p = 1 атм, t = 25°C)?

14. Каков объем 1 г кислорода при нормальных условиях (p = 1 атм,

t = 25°C)?

15. Каков объем 1 г азота при нормальных условиях (p = 1 атм, t = 25°C)?

16. Каков объем 1 г углекислого газа при нормальных условиях (p = 1 атм, t = 25°C)?

17. Каков объем 1 г гелия при нормальных условиях (p = 1 атм, t = 25°C)?

18. Каков объем 1 г водяного пара при нормальных условиях (p = 1 атм,

t = 25°C)?

19. Каков вес 1 л водорода при нормальных условиях (p = 1 атм, t = 25°C)?

20. Каков вес 1 л кислорода при нормальных условиях (p = 1 атм,

t = 25°C)?

21. Каков вес 1 л азота при нормальных условиях (p = 1 атм, t = 25°C)?

22. Каков вес 1 л углекислого газа при нормальных условиях (p = 1 атм,

t = 25°C)?

23. Каков вес 1 л гелия при нормальных условиях (p = 1 атм, t = 25°C)?

24. Каков вес 1 л водяного пара при нормальных условиях (p = 1 атм,

t = 25°C)?

25. Каков (в литрах) объем одного моля водорода при нормальных условиях (p = 1 атм, t = 25°C)?

26. Каков (в литрах) объем одного моля кислорода при нормальных условиях (p = 1 атм, t = 25°C)?

27. Каков (в литрах) объем одного моля азота при нормальных условиях (p = 1 атм, t = 25°C)?

28. Каков (в литрах) объем одного моля углекислого газа при нормальных условиях (p = 1 атм, t = 25°C)?

29. Каков (в литрах) объем одного моля гелия при нормальных условиях (p = 1 атм, t = 25°C)?

30. Каков (в литрах) объем одного моля водяного пара при нормальных условиях (p = 1 атм, t = 25°C)?

31. Какова температура 1 г водорода, если он занимает объем 1 л при нормальном давлении (p = 1 атм)?

32. Какова температура 1 г кислорода, если он занимает объем 1 л при нормальном давлении (p = 1 атм)?



33. Какова температура 1 г азота, если он занимает объем 1 л при нормальном давлении (p = 1 атм)?

34. Какова температура 1 г водяного пара, если он занимает объем 1 л при нормальном давлении (p = 1 атм)?

35. Какова температура 1 г гелия, если он занимает объем 1 л при нормальном давлении (p = 1 атм)?

36. Какова температура 1 г углекислого газа, если он занимает объем 1 л при нормальном давлении (p = 1 атм)?

37. Каково давление 1 г водорода, если он занимает объем 1 л при нормальной температуре (t = 25°C)?

38. Каково давление 1 г кислорода, если он занимает объем 1 л при нормальной температуре (t = 25°C)?

39. Каково давление 1 г азота, если он занимает объем 1 л при нормальной температуре (t = 25°C)?

40. Каково давление 1 г водяного пара, если он занимает объем 1 л при нормальной температуре (t = 25°С)?

41. Каково давление 1 г гелия, если он занимает объем 1 л при нормальной температуре (t = 25°С)?

42. Каково давление 1 г углекислого газа, если он занимает объем 1 л при нормальной температуре (t = 25°С)?

43. В запаянном баллоне находится водород. При температуре 25°C его давление – 2 атм. Каково будет давление при температуре 250°C?

44. В запаянном баллоне находится кислород. При температуре 25°C его давление – 2 атм. Каково будет давление при температуре 250°C?

45. В запаянном баллоне находится азот. При температуре 25°C его давление – 2 атм. Каково будет давление при температуре 250°C?

46. В запаянном баллоне находится углекислый газ. При температуре 25°C его давление – 2 атм. Каково будет давление при температуре 250°C?

47. В запаянном баллоне находится гелий. При температуре 25°C его давление – 2 атм. Каково будет давление при температуре 250°C?

48. В теплоизолированном (при исходной температуре 25°C) цилиндре объемом 4 л под поршнем площадью 20 см2 и массой 5 кг находится водород. На поршень поставили гирю массой 5 кг. Каков станет объем газа?

49. В теплоизолированном (при исходной температуре 25°C) цилиндре объемом 4 л под поршнем площадью 20 см2 и массой 5 кг находится кислород. На поршень поставили гирю массой 5 кг. Каков станет объем газа?

50. В теплоизолированном (при исходной температуре 25°C) цилиндре объемом 4 л под поршнем площадью 20 см2 и массой 5 кг находится азот. На поршень поставили гирю массой 5 кг. Каков станет объем газа?

51. В теплоизолированном (при исходной температуре 25°C) цилиндре объемом 4 л под поршнем площадью 20 см2 и массой 5 кг находится углекислый газ. На поршень поставили гирю массой 5 кг. Каков станет объем газа?

52. В теплоизолированном (при исходной температуре 25°C) цилиндре объемом 4 л под поршнем площадью 20 см2 и массой 5 кг находится гелий. На поршень поставили гирю массой 5 кг. Каков станет объем газа?

53. В теплоизолированном (при исходной температуре 25°C) цилиндре объемом 4 л под поршнем площадью 20 см2 и массой 5 кг находится водород. На поршень поставили гирю массой 5 кг. Какой после этого станет температура газа?

54. В теплоизолированном (при исходной температуре 25°C) цилиндре объемом 4 л под поршнем площадью 20 см2 и массой 5 кг находится кислород. На поршень поставили гирю массой 5 кг. Какой после этого станет температура газа?

55. В теплоизолированном (при исходной температуре 25°C) цилиндре объемом 4 л под поршнем площадью 20 см2 и массой 5 кг находится азот. На поршень поставили гирю массой 5 кг. Какой после этого станет температура газа?

56. В теплоизолированном (при исходной температуре 25°C) цилиндре объемом 4 л под поршнем площадью 20 см2 и массой 5 кг находится углекислый газ. На поршень поставили гирю массой 5 кг. Какой после этого станет температура газа?

57. В теплоизолированном (при исходной температуре 25°C) цилиндре объемом 4 л под поршнем площадью 20 см2 и массой 5 кг находится гелий. На поршень поставили гирю массой 5 кг. Какой после этого станет температура газа?

58. Объем пузырька воздуха по мере всплывания его со дна озера на поверхность увеличивается в 3 раза. Какова глубина озера?

59. Определить плотность смеси, состоящей из 4 г водорода и 32 г кислорода, при температуре 7°C и давлении 93 кПа.

60. Некоторый идеальный газ испытывает сначала изобарное расширение, а затем изотермическое сжатие. Изобразить эти процессы в координатах p, T и V, T.

61. Изобразите графически зависимость плотности некоторой массы идеального газа от давления p при изотермическом процессе.

62. Некоторая масса идеального газа изобарно нагревается, а затем после изотермического сжатия и изохорного охлаждения возвращается в исходное состояние. Изобразить эти процессы в координатах p, T и p, V.

63. Сколько молекул воздуха находится в комнате объемом 240 м3 при температуре 15°C и давлении 105 Па?

64. При постоянной температуре концентрация молекул газа уменьшилась в два раза. Как при этом изменились давление и объем? Рисунок 1

65. На рисунке 1 в координатах р, Т представлен замкнутый цикл. Представить этот цикл в координатах V, Т (р – давление, V – объем, Т – температура).

66. Имеется сосуд объемом 0,5 м3 и поршневой насос с объемом 50 см3. Сколько качаний нужно сделать, чтобы давление в сосуде уменьшилось от 760 мм рт.ст. до 700 мм рт.ст. Считать, что процесс происходит при постоянной температуре.

67. На рисунке 2 в координатах V,T представлен циклический процесс, проведенный с неизменной массой идеального газа. Представить этот процесс в координатах р,T (V-объем, T-температура, р — давление газа). Рисунок 2

68. Сколько молей воды содержится в ведре вместимостью 9 л, если молярная масса воды 0,018 кг/моль. Плотность воды 103 кг/м3.

69. Предельно допустимая концентрация молекул паров ртути в воздухе равна 3×1016 м-3. При какой массе паров ртути в 1 м3воздуха появляется опасность отравления?

70. Баллон емкостью 10 л содержит 2 г водорода и 14 г азота при температуре 25˚С. Каково будет давление в баллоне, если в него дополнительно накачать 44 г углекислого газа? Температура при накачивании повысилась на 2˚ С.

71. При постоянной температуре газ сжали от объема 0,15 м3до 0,1 м3. Давление газа при этом повысилось на 20 Н/см2. Определить начальное давление газа.

72. В баллоне емкостью 60 л находится 265 г газа при температуре 0°С и давлении 5 МПа. Какой это газ?

73. Идеальный одноатомный газ совершает замкнутый цикл, состоящий из двух изобар и двух изохор (рисунок 3), причем наибольшее давление газа в два раза больше наименьшего, а наибольший объем в 4 раза больше наименьшего. Определить КПД цикла.

Рисунок 3

74. Теплоизолированный сосуд разделен пористой перегородкой на две равные части. Атомы гелия могут свободно проникать через поры в перегородке, а атомы аргона – нет. В начальный момент в одной части сосуда находится 8 г гелия, а в другой – 40 г аргона. Температура гелия 270С, а температура аргона 3270 С. Определить температуру гелия после установления равновесия в системе.

75. При увеличении абсолютной температуры идеального газа в 4 раза давление газа увеличилось на 50 %. Как и во сколько раз изменился объем? Масса газа постоянна

76. На рисунке 4 представлен циклический процесс, проведенный с неизменной массой идеального газа. Представьте этот процесс в координатах V,T и p,V, где V — объем, T — температура, p — давление газа. Рисунок 4

77. На диаграмме p,V (рисунок 5) изображены изотермы трех газов — кислорода, гелия и углекислого газа. Массы газов одинаковы. Измерения производились при одной и той же температуре. Какая изотерма соответствует какому газу?

Рисунок 5

78. Температура воздуха в комнате изменилась от 7°С до 27° С. На сколько процентов уменьшилось число молекул в комнате?

79. Сколько молекул содержится в 1 л воды и какой объем при нормальных условиях занимает водяной пар той же массы (водяной пар считать идеальным газом)?

80. Найти молярную массу смеси, состоящей из 20 г кислорода, 30 г азота и 60 г окиси углерода.

81. Какой из циклов, изображенных на рисунке 6, соответствует циклу в координатах p,V и состоящему из двух изохор и двух изобар?

1) 2) 3)

4) 5)

 

Рисунок 6

 

82. Какой из перечисленных газов: водород, гелий, кислород, азот в количестве одного моля нужно взять, чтобы создать максимальное давление, одинаковое для всех газов, в сосуде объемом 1 м3 при 273 К?

 

83. В баллоне находится 2,2 моля газа. Сколько молекул находится в баллоне?

84. Идеальный газ сначала нагревался при постоянном давлении, потом его давление увеличивалось при постоянном объеме, затем при постоянной температуре давление его уменьшилось до первоначального значения. Начертить графики этих процессов в координатах p-T, p-V, V-T.

85. Начертите графики изменения плотности идеального газа при изотермическом и изобарическом процессах в координатах плотность-температура.

86. Из баллона с кислородом, находящимся при высоком давлении P, очень медленно выпустили половину массы газа. Как изменились при этом давление и температура? Что изменится и почему, если этот процесс совершать очень быстро?

87. При нагревании газа получена графическая зависимость давления от температуры (рисунок 7). Определить по графику, сжимался или расширялся газ во время нагревания?

Рисунок 7

88. В баллоне находится 0,01 моль газа. Сколько молекул находится в баллоне?

89. На рисунке 8 представлен график изменения состояния идеального газа в координатах p, T. Представьте этот процесс графически в координатах T, V и p, V (p – давление, V – объем, T – температура газа) Рисунок 8

90. Какие процессы представлены на рисунке 9? Изобразите их в координатах p-V м V-T

Рисунок 9

91. На рисунке 10 изображены зависимости давления от температуры для одной и той же массы газа, помещенного в сосуды различных объемов. Какой из графиков соответствует изменению состояния газа, помещенного в меньший объем?

Рисунок 10

92. Найти число молекул в 1 см3 и плотность водорода при давлении 10-6 мм. рт. ст. и температуре 17°С.

93. Как и во сколько раз изменится число молекул газа, заключенных в единице объема, если давление газа возрастет с 1 атм. до 3 атм., а температура повысится с 0°С до 273°С?

94. Определить число молей и число молекул кислорода, масса которого 0,5 кг.

95. Баллон емкостью 12 л содержит углекислый газ. Давление газа 9 атм., температура 15°С. Определить массу газа и массу молекулы газа.

96. Сколько киломолей и сколько молекул газа находится в колбе 240 см3, если температура газа 20°С и давление 750 мм рт. ст.?

97. Найти число атомов, содержащихся в капельке ртути массой 2 г. Какова масса атома ртути?

98. Во фляжке емкостью 0,5 л находится 0,3 л воды. Турист пьет из нее воду, плотно прижав губы к горлышку так, что во фляжку не попадает наружный воздух. Сколько воды удается выпить туристу, если он может понизить давление оставшегося во фляжке воздуха до 80 кПа? Атмосферное давление считать равным 100 кПа.

99. В сосуде, объем которого 2 л, находится 0,2 моля кислорода. Определить концентрацию молекул газа.

100. В цилиндр, длиной 1,6 м, заполненный воздухом при нормальном давлении, начали медленно вдвигать поршень, площадь основания которого 200 см2. Определить силу, действующую на поршень, если его остановить на расстоянии 10 см от дна цилиндра.

101. В воде на глубине 100 м находится шарообразный воздушный пузырь. На какой глубине пузырь должен расшириться в шар вдвое большего радиуса? Сил поверхностного натяжения не учитывать. Ускорение свободного падения принять равным 10 м/с2.

102. Плотность серебра 15,5 г /см3. Сколько атомов серебра содержится в 1 мм3?

103. Баллон емкостью 10 л содержит 2 г водорода и 14 г азота при температуре 25°С. Каково будет давление в баллоне, если в него дополнительно накачать 44 г углекислого газа? Температура при накачивании повысилась на 2°С.

104. В одном баллоне емкостью 15 дм3 находится газ под давлением 0,2 МПа, а в другом — тот же газ под давлением 1 МПа. Баллоны соединены трубкой с краном. Если открыть кран, то в обоих баллонах устанавливается давление 0,4 МПа. Какова емкость второго баллона? Температура газа в баллонах одинакова.

105. Колба емкостью 0,5 л содержит азот при нормальных условиях. Сколько молекул газа находится в колбе?

106. Баллон емкостью 30 л содержит смесь водорода и гелия при температуре 300 К и давлении 0,8 МПа. Масса смеси 24 г. Определить массу водорода и массу гелия.

107. В закрытом сосуде емкостью 2 м3 находится 1,4 кг азота и 2 кг кислорода. Найти давление газовой смеси в сосуде, если температура смеси 27°С. Определить массу молекул азота и кислорода.

108. Определить плотность смеси 4 г азота и 32 г кислорода при температуре 70С и давление 700 мм рт.ст.

109. В сосуде находится газ под давлением 0,15 МПа при температуре 2730 С. Какое количество молекул находится в единице объема сосуда?

110. Найти массу одного атома водорода и число атомов, содержащихся в одном грамме водорода

111. В сосуде, при температуре 1000С и давлении 4∙105 Па, находится 2 м3 в смеси кислорода О2 и сернистого газа SO2. Определить парциальное давление компонентов, если масса сернистого газа 8 кг.

112. Найти число молекул в 1 см3 и плотность азота при давлении 10-6 мм рт. ст. и температуре 15º С.

113. В двух сосудах емкостью по 5 л находится соответственно азот под давлением 1 атм. И окись углерода под давлением 5 атм. Сосуды соединяют тонкой трубкой, объемом которой можно пренебречь. Найти установившееся давление смеси, если начальная температура обоих газов равна температуре окружающей среды.

114. Найти объем, занимаемый смесью 2,8 кг азота и 3,2 кг кислорода при температуре 170 С и давлении 4∙105 Н/мв2.

115. Найти массу молекулы азота и число молекул, содержащихся в одном грамме азота

116. Найти число атомов, содержащихся в капельке ртути массой 1 г, и массу атома ртути.

117. Найти плотность воздуха при температуре 270 С и давлении 1013 гПа. Считать то воздух состоит (по массе) из 75,4 % азота, 23,2% кислорода, 1,4 % аргона. Содержанием остальных газов пренебречь.

118. Плотность смеси азота и водорода при температуре 47 0С и давлении 2 атм. равна 0,30 г/л. Найти концентрацию молекул азота и водорода в смеси.

119. Сколько метана может вместить баллон емкостью 15 дм3 при давлении 106 Па и температуре 270 С? Чему равна масса киломоля и молекулы метана?

120. Газ был сжат адиабатически так, что его объем уменьшился в 8 раз. При этом температура газа возросла 27 0С до 410,3 0С. Сколько атомов содержит молекула данного газа?

121. Определить плотность смеси газов, находящихся при давлении 1 МПа и температуре 270С. Смесь состоит из 5 кмолей азота, 1,5 кмоля кислорода и 0,5 кмоля углекислого газа.

122. В сосуде объемом 10 л находится 2∙1023 молекул газа. Какова температура газа, если давление в сосуде равно 0,12 МПа?

123. В баллоне находилось 2 кг газообразного водорода. В баллон дополнительно накачали 8 кг метана. Во сколько раз изменилось давление в баллоне, если температура газов осталось неизменной?

124. При какой температуре кислород, находясь под давлением 0,2 МПа, имеет плотность 1,2 кг/м3. Какова при этом концентрация молекул кислорода?

125. Какое количество кислорода выпустили из баллона емкостью 10 л, если при этом показания манометра на баллоне изменились от 14 до атм., а температура понизилась от 27 0С до 7 0С?

126. В сосуде при t0С = 270 С и давлении Р = 0,1 МПа содержится смесь газов- кислорода массой m1 = 8 г азота массой m2 = 14 г. Определить плотность смеси.

127. Наименьший объем V1 газа, совершающий цикл Карно, равен 30,6 л. Определить наибольший объем V3 , если объем V2 в конце изотермического расширения и объем V4 в конце изотермического сжатия равны соответственно 120 л и 37,8 л.

128. Один моль идеального двухатомного газа совершает цикл, состоящий из двух изохор и двух изобар. Наименьший объем V1=10 л, наибольший объем V2=20 л, наименьшее давление p1=246 кПа, наибольшее p2=410 кПа. Построить график цикла. Определить температуру Т газа для характерных точек цикла. Определить температуру Т газа для характерных точек и работу цикла.

129. Сколько молей и сколько молекул газа находится в колбе емкостью 480 см3,если температура газа 200С и давление 380 мм. рт. ст.?

130. Колба емкостью 1,5 л содержит газ при нормальных условиях. Сколько молекул газа находится в колбе. Какова масса газа, если это: водород, кислород, азот?

131. В колбе емкостью 500 см3 содержится некоторой газ при температуре Т=300 К. На сколько понизится давление p газа в колбе, если вследствие утечки из колбы выйдет N = 25×1020 молекул?

132. Взрослый человек выдыхает в течение суток приблизительно 0,65 м3 углекислого газа (предполагается, что газ приведен к нормальным условиям). Найти массу чистого углерода, выдыхаемого человеком. Каков был бы объем кубика, содержащего такую же массу? Сколько в нем содержалось бы атомов углерода?

133. В воде на глубине 0,9 м находится шарообразный воздушный пузырь. На какой глубине пузырь должен расширится в шар в 1,5 раза большего радиуса? Сил поверхностного натяжения не учитывать.

134. Ксенон составляет 48∙10-5 % (по массе) атмосферного воздуха. Сколько атомов ксенона находится в 1 см3 воздуха?

135. Плотность серебра 10,5 г/см3. Сколько атомов серебра содержится в 1 мм3?

136. Плотность пара некоторого соединения углерода с водородом равна 3 г/л при 43 0С и 820 мм рт.ст. Какова химическая формула этого соединения?

137. Газ, находящийся при нормальных условиях, имеет объем 44,8 л. Масса газа 32 г. Молекулы газа состоят из атомов углерода и водорода. Какой это газ?

138. Какое давление рабочей смеси установилось в цилиндре двигателя внутреннего сгорания, если к концу такта сжатия температура повысилась с 470С до 3670С, а объем уменьшился с 1,8 до 0,3 л? Первоначальное давление было 0,1 МПа.

139. Сколько молекул водорода находится в объеме 1,55 л температуре 270С и давлении 570 мм рт. ст.?

140. В баллоне емкостью 2 м3 содержится смесь азота N2 и окиси азота NО. Определить массу окиси азота, если масса смеси равна 14 кг, температура 300 К и давлении 0,6×106 Па.

141. Используя формулы молекулярно-кинетической теории, вычислить удельные теплоемкости аргона, азота, метана.

142. Плотность газа при давлении 720 мм рт. ст. и температуре 00С равна 1,35 г/л. Найти массу киломоля газа и массу молекулы.

143. Два одинаковых по объему баллона соединены трубкой с краном. В одном баллоне находится 32 г метана под давлением 0,5 МПа, в другом смесь 32 г кислорода и 84 г азота. Каково давление смеси? Какое давление установится в баллонах, если открыт кран? Температура всех газов одинакова.

 

IV.1 Физические основы термодинамики. Первое начало термодинамики

1 Внутренняя энергия(U)термодинамической системы –это энергия хаотического (теплового) движения микрочастиц системы (молекул, атомов, электронов, ядер и т.д.) и энергия взаимодействия этих частиц.

– внутренняя энергия идеального газа, где i – число степеней свободы молекулы данного газа.

2 Количество теплотыQ – (мера процесса теплообмена, зависит от вида процесса). Q > 0 при получении энергии системой; Q < 0 при отдаче энергии системой.

3 Теплоемкость – (зависит от вида процесса) .

С – молярная теплоемкость; с – удельная теплоемкость. .

Связь молярной и удельной теплоемкостей

;

4 Работа в термодинамике А – (зависит от вида процесса).

А> 0 при совершении работы самой системой;

A < 0 при совершении работы внешними силами над системой;

работа расширения идеального газа.

преобразование температурных шкал — WebMath

Быстро! Мне нужна помощь с: Выберите элемент справки по математике … Исчисление, Производные вычисления, Интеграционное вычисление, Частное правило, Монеты, Подсчет комбинаций, Поиск всех комплексных чисел, Сложение комплексных чисел, Вычисление с комплексными числами, Умножение комплексных чисел, Степени комплексных чисел, Преобразование вычитания, Преобразование площади, Преобразование скорости, Преобразование длины , VolumeData Analysis, Find the AverageData Analysis, Find the Standard DeviationData Analysis, HistogramsDecimals, Convert to a дробь, Электричество, Стоимость разложения, IntegerFactors, Greatest CommonFactors, Least CommonFractions, AddingFractions, ComparingFractions, ConvertingFractions, Convert to a decimalFractions, DécimalFractions, Convert to a decimalFractions ВычитаниеФракции, Что это такое: Геометрия, Коробки, Геометрия, Круги, Геометрия, Цилиндры, Геометрия, Прямоугольники, Геометрия, Правые треугольники, Геометрия, Сферы, Геометрия, Квадраты, Графики, Линии, Графики, Любая функция, Графики, Круги hing, EllipsesGraphing, HyperbolasGraphing, InequalitiesGraphing, Polar PlotGraphing, (x, y) pointInequalities, GraphingInequalities, SolvingInterest, CompoundInterest, SimpleLines, Equation from point and slopeLines, The Equation from slopeLinesLines Theotation, The Equation from slopeLines Theotation и Y-intation , Нахождение шансовМатематика, Практика многочленов по математике, Практика основМетрическая система, Преобразование чисел, Сложение чисел, Вычисление с числами, Вычисление с переменными числами, Деление чисел, Умножение чисел, Сравнение числовых линий, Числовые строки, Разместите значения чисел, Произношение чисел, Округление чисел, Вычитание числа слагаемых, Вычитание числа Квадратные многочлены, Деление многочленов, Факторизация разности квадратов многочленов, Факторизация триномов многочленов, Факторинг с GCF Полиномы, Умножение многочленов, Возведение в степеньПрактика, Математические задачиПропорции, Квадратные уравнения ormulaQuadratic Equations, Solve by FactoringRadicals, Other RootsRadicals, Square RootsRatios, Что они представляют собой Устранение, Экономия на продажной цене, РасчетНаучная нотация, ПреобразованиеНаучной нотации, ДелениеНаучная нотация, УмножениеФормы, ПрямоугольникиУпрощение, Упрощение, Упрощение продуктов, Упрощение, Упрощение, Упрощение, Упрощение, Упрощение, Упрощение продуктов , Правые треугольники, Ветер, Рисунок

Калькулятор точки росы

Калькулятор рассчитывает температуру, до которой воздух должен быть охлажден, чтобы он стал насыщенным водяным паром и образовал росу.

Укажите любых двух из трех нижеприведенных переменных для расчета третьей.


Калькулятор охлаждения связанного ветра | Калькулятор теплового индекса

Что такое влажность?

Влажность определяется как количество водяного пара (газообразная фаза воды) в воздухе. Это индикатор наличия росы, мороза, тумана и осадков. Максимальное количество водяного пара, которое может удерживать воздух, зависит от температуры; чем выше температура, тем большее количество водяного пара он может удержать, прежде чем достигнет насыщения.

Влажность часто называют абсолютной влажностью и относительной влажностью, как в этом калькуляторе. Значение абсолютной влажности возвращается как часть результатов расчета, но именно относительная влажность широко используется в повседневной жизни и используется как часть расчета температуры точки росы.

Абсолютная влажность — это измерение содержания воды в воздухе, обычно в граммах на кубический метр. Он рассчитывается путем деления общей массы водяного пара на объем воздуха.Учитывая такое же количество водяного пара в воздухе, абсолютная влажность не меняется с температурой при фиксированном объеме. Если объем не фиксирован, как в атмосфере, абсолютная влажность изменяется в ответ на изменения объема, вызванные колебаниями температуры и давления.

Относительная влажность сравнивает текущее отношение абсолютной влажности к максимальной влажности для данной температуры и выражает это значение в процентах. Чем выше процент, тем выше влажность.На него влияют как температура, так и давление. При таком же количестве водяного пара в прохладном воздухе будет более высокая относительная влажность, чем в более теплом.

Относительная влажность — широко используемый показатель в сводках погоды и прогнозах погоды и является хорошим индикатором осадков, росы, мороза, тумана и видимой температуры. Кажущаяся температура — это температура, воспринимаемая людьми. Летом, чем выше относительная влажность, тем выше кажущаяся температура. Это результат более высокой влажности, что снижает скорость испарения пота, что увеличивает воспринимаемую температуру.

Относительная влажность 100% означает, что воздух насыщен, а это означает, что при текущих условиях водяной пар в воздухе не может увеличиваться дальше в нормальных условиях. Относительная влажность 100% также является точкой, при которой может образовываться роса.

Что такое точка росы?

Точка росы определяется как температура, при которой данный объем воздуха при определенном атмосферном давлении насыщается водяным паром, вызывая конденсацию и образование росы. Роса — это конденсированная вода, которую человек часто видит рано утром на цветах и ​​траве.Точка росы варьируется в зависимости от количества водяного пара, присутствующего в воздухе, при этом более влажный воздух приводит к более высокой точке росы, чем сухой воздух. Кроме того, чем выше относительная влажность, тем ближе точка росы к текущей температуре воздуха, а относительная влажность 100% означает, что точка росы эквивалентна текущей температуре. В случаях, когда точка росы ниже точки замерзания (0 ° C или 32 ° F), водяной пар превращается непосредственно в иней, а не в росу.

В то время как восприятие различается у разных людей, и люди на определенном уровне могут адаптироваться к более высоким точкам росы, более высокие точки росы обычно вызывают дискомфорт, потому что влажность препятствует правильному испарению пота, затрудняя охлаждение тела человека.И наоборот, более низкие точки росы также могут быть неудобными, вызывая раздражение и растрескивание кожи, а также высушивая дыхательные пути человека. Управление по охране труда и здоровья США рекомендует поддерживать температуру воздуха в помещении в пределах 68–76 ° F при относительной влажности 20–60%.

Точка росы также учитывается в авиации общего назначения для расчета вероятности таких потенциальных проблем, как обледенение карбюратора или туман. В некоторых случаях устройства, известные как измерители точки росы, используются для измерения точки росы в широком диапазоне температур.Эти устройства состоят из полированного металлического зеркала, которое охлаждается при прохождении через него воздуха. Температура, при которой на зеркале образуется роса, и есть точка росы.

Калькулятор плотности воздуха

— Что такое плотность воздуха?

Определение плотности воздуха — какова формула плотности воздуха?

Основное определение плотности воздуха очень похоже на общее определение плотности. Он говорит нам, сколько весит определенный объем воздуха. Мы можем выразить это следующей формулой плотности воздуха:

ρ = масса воздуха / объем

Из приведенного выше уравнения можно предположить, что плотность воздуха — это постоянная величина, которая описывает определенное свойство газа.Однако плотность каждого вещества (твердых тел, жидкостей, газов) зависит, сильнее или слабее, не только химического состава вещества , но также и от внешних условий , таких как давление и температура.

Из-за этих зависимостей и того факта, что атмосфера Земли содержит различные газы (в основном азот, кислород, аргон и водяной пар ), определение плотности воздуха требует дальнейшего расширения. В наш калькулятор плотности воздуха была внесена соответствующая модификация: формула плотности воздуха указана в разделе «Как рассчитать плотность воздуха?».

Кстати, хотелось бы поднять интересный момент. Что вы думаете? Влажный воздух тяжелее или легче сухого? Правильный ответ может быть не таким интуитивным, как вы думаете вначале. Фактически, : чем больше водяного пара мы добавляем в воздух, тем менее плотным он становится! Возможно, вам трудно поверить в это, но мы попытаемся убедить вас несколькими логическими аргументами.

Прежде всего, нам нужно обратиться к закону Авогадро , который гласит, что

равные объемы всех газов, при одинаковой температуре и давлении, имеют одинаковое количество молекул.

Представьте, что вы помещаете сухой воздух в контейнер с фиксированным объемом, температурой и давлением. В состав идеально сухого воздуха входят:

  • 78% молекул азота N₂ с двумя атомами N с атомным весом 14 u (общий вес 28 u),
  • 21% молекул кислорода O₂ , который имеет два атома O с атомной массой 16 u (общий вес 32 u), и
  • 1% молекул аргона Ar ( Ar имеет один атом с атомной массой 39.8 ед).

Обратите внимание, что каждая указанная молекула тяжелее или равна 18 ед. Теперь добавим в газ несколько молекул водяного пара с общим атомным весом 18 u ( H₂O — два атома водорода 1 u и один кислород 16 u). Согласно закону Авогадро, общее количество молекул в емкости остается неизменным при тех же условиях (объем, давление, температура). Это означает, что молекул водяного пара должны заменить азота, кислорода или аргона. Поскольку молекулы H₂O легче других газов, общая масса газа уменьшается, что также снижает плотность воздуха.

Как меняется температура днем ​​и ночью

Когда я учился в школе, казалось, что на мне никогда не было подходящего пальто. Если бы я шел в школу в 7:30 (07:30) в своем тяжелом пальто, мне часто было бы слишком жарко по дороге домой в 15:30. (1530) С другой стороны, утром было бы слишком холодно, чтобы надеть более светлое пальто.

Теперь, как опытный метеоролог, я знаю причины, почему. Вы знаете, в какое время суток самое холодное? Или когда он самый теплый?

К счастью, довольно легко найти данные, чтобы ответить на этот вопрос.На веб-сайте GLOBE вы можете найти GLOBE ONE в разделе «проекты» и найти данные для 10 автоматических метеостанций из округа Блэк-Хок, штат Айова. На рис. 1 показано изменение температуры в течение пяти дней с хорошей погодой в апреле 2002 г. на станции 4.

Рис. 1. Температура воздуха Tavg (красный) и точка росы Tdavg (синий) на участке в округе Блэк-Хок, штат Айова. Высота: 1,5 м над поверхностью. Это средние данные для пяти дней с ясным небом в апреле 2004 года.

Если посмотреть на график, самая высокая температура приходится на 22:30 UTC или 4:30 (16:30) днем ​​по местному стандартному времени.Самая низкая температура — около 7 часов утра по местному времени.

Вы ожидали, что самая высокая температура будет в полдень, когда солнце находится выше всего в небе? Многие люди так делают. Почему этого не происходит?

Давайте начнем с рассмотрения энергии, исходящей от Солнца. Между восходом и заходом солнца солнечное излучение постоянно добавляет энергии поверхности Земли. Если бы эта энергия каким-то образом не ускользнула, температура была бы самой высокой на закате.

Мы знаем, что этого не происходит.Итак, давайте внимательнее посмотрим, что происходит. Я буду использовать данные из юго-восточного Канзаса.

Рис. 2. Для двух дней без ясного неба на лугах на юго-востоке Канзаса, температура поверхности земли и температура воздуха (вверху), нисходящая (вниз) солнечная радиация и чистая радиация (внизу). Обратите внимание, как чистая радиация падает до нуля примерно в 19 часов после полуночи и остается отрицательной примерно до 5 часов после полуночи. Время указано местное.

На рисунке 2, как и на рисунке 1, температура воздуха достигает максимума ближе к вечеру; в 16 часов после полуночи (16:00 или 4 р.м. по местному стандартному времени) 30 мая и в 16 часов после полуночи 31 мая (40 минус 24 часа = 16 часов, 16:00 или 16:00).

Мы знаем, что воздух на высоте 1,5 метра нагревается за счет излучения и конвекции.

Внизу рисунка 2 показано, что происходит с излучением. В 16:00 от Солнца все еще поступает энергия. (16:00) и позже (примерно до 19,5 часов после полуночи). Однако нисходящая солнечная радиация — это еще не все.

Часть солнечной энергии отражается вверх.

Также воздух (парниковые газы), облака и поверхность Земли излучают энергию в инфракрасном диапазоне. В дни, представленные на рисунках 1 и 2, облака, конечно, не имеют значения. Обычно инфракрасное излучение от земли больше, чем от воздуха. Инфракрасное излучение поверхности — это то, что измеряется прибором, используемым в протоколе GLOBE Surface Temperature Protocol: прибор преобразует инфракрасное излучение от поверхности (травы, асфальта или голой земли) в температуру.(Для получения дополнительной информации о температуре поверхности см. «Руководство / протоколы для учителя» в раскрывающемся меню «Учителя».)

Если сложить все инфракрасное излучение, чистое инфракрасное излучение направлено вверх (апвеллинг).

Чистая радиация на Рисунке 2 — это приходящая радиация (нисходящая солнечная и инфракрасная) за вычетом исходящей радиации (отраженная солнечная и восходящая инфракрасная). То есть чистая радиация снижается между пятью часами после полуночи (05:00) и 19 часами после полуночи (19:00 или 7 баллов).м.).

Думаю, я убедил вас (и себя), почему самая теплая температура воздуха не тогда, когда наиболее сильный солнечный свет. Но почему не самая высокая температура около 19 часов после полуночи, когда чистая радиация перестает нагревать землю и начинает становиться отрицательной?

Причина в том, что тепло теряется из-за конвекции.

Воздушные потоки уносят тепло от поверхности. Судя по всему, в 16:00. (1600) по местному времени в оба дня на Рисунке 2, входящая энергия от чистого излучения просто уравновешивает чистую исходящую энергию от конвекции (конвекция переносит тепло от земли на 1.5 метров, но он также переносит тепло с высоты 1,5 метра), а температура воздуха достигает максимума. До 16:00 (16:00) чистая радиация приносит больше энергии, чем выводят конвекционные потоки, и температура воздуха повышается. После 16:00 (16:00) конвекция уносит больше тепла, чем приносит излучение, и температура снижается.

Иногда мы называем суммирование входящего и исходящего тепла «тепловым бюджетом» из-за его сходства с деньгами. Когда вы откладываете больше денег, чем тратите, сумма денег на вашем банковском счете — или в вашей копилке — увеличивается.Если вы тратите больше денег, чем откладываете, сумма денег на вашем банковском счете или в копилке становится меньше. Когда вы тратите столько, сколько вкладываете, сумма денег не меняется.

А как насчет температуры поверхности? Это немного сложнее, потому что земля не только теряет энергию из-за конвекционных потоков, но также теряет энергию из-за испарения и нагрева более холодной почвы под ней. Эти дополнительные потери приводят к понижению температуры поверхности днем ​​раньше, чем температура воздуха, примерно на 14 часов после полуночи.

Ночью все в некотором роде проще. Нет солнечного света. В ясные ночи с небольшим ветром, например, показанные на рисунках 1 и 2, воздух и земля продолжают охлаждаться, испуская инфракрасное излучение (обратите внимание, что чистое излучение в нижней части рисунка 2 отрицательно в течение ночи). Так как это продолжается всю ночь, самые низкие температуры бывают ранним утром, ближе к времени восхода солнца.

Передача тепла по воздуху (конвекция) происходит, когда ветер поднимает воздух у поверхности.Это усложняет ситуацию. В среднем конвекция имеет тенденцию замедлять падение температуры на 1,5 метра с минимумом около восхода солнца.

Холодная среда — Работа в холоде: OSH Answers

Одежда

Защитная одежда необходима для работы при температуре 4 ° C или ниже. Одежда должна подбираться в соответствии с температурой, погодными условиями (например, скоростью ветра, дождя), уровнем и продолжительностью активности и дизайном работы. Эти факторы важно учитывать, чтобы вы могли регулировать количество тепла и потоотделения, выделяемого во время работы.При слишком быстром темпе работы или неправильном выборе типа и количества одежды может возникнуть чрезмерное потоотделение. Одежда рядом с телом станет влажной, и ее теплоизоляционные свойства резко снизятся. Такое снижение изоляционных свойств увеличивает риск травм от переохлаждения.

  • Одежду следует носить в несколько слоев, которые обеспечивают лучшую защиту, чем одна толстая одежда. Воздух между слоями одежды обеспечивает лучшую изоляцию, чем сама одежда.Наличие нескольких слоев также дает вам возможность открыть или удалить слой, прежде чем вы слишком сильно нагреетесь и начнете потеть, или добавить слой, когда вы сделаете перерыв. Это также позволяет вам приспособиться к уровню активности, изменению температуры и погодных условий. Последовательные внешние слои должны быть больше, чем внутренний слой, в противном случае самый внешний слой будет сжимать внутренние слои и уменьшать изоляционные свойства одежды.
  • Внутренний слой должен обеспечивать изоляцию и отводить влагу от кожи, чтобы она оставалась сухой.Для этого подойдет термобелье из полиэстера или полипропилена. Полипропилен отводит пот от кожи. Он также защищает второй слой от кожи.
  • Дополнительные слои одежды должны обеспечивать достаточную изоляцию для погодных условий, в которых выполняется работа. Они также должны легко открываться или сниматься, прежде чем вы слишком сильно нагреетесь, чтобы предотвратить чрезмерное потоотделение во время напряженной деятельности. На внешней куртке должны быть средства для закрытия и открытия талии, шеи и запястий, чтобы контролировать, сколько тепла сохраняется или отводится.У некоторых курток есть сетчатые карманы и вентиляционные отверстия вокруг туловища и под мышками (на молниях или застежках-липучках) для дополнительной вентиляции.
  • Для работы во влажных условиях верхний слой одежды должен быть водонепроницаемым.
  • Если рабочая зона не может быть защищена от ветра, следует использовать легкосъемную ветрозащитную одежду.
  • При очень холодных условиях необходимо иметь в наличии защитную одежду с подогревом, если работа не может быть выполнена в более теплый день.
  • Носите шляпу, соответствующую условиям, в том числе согревая уши. Если требуется каска, вязаная шапка или подкладка под каску могут уменьшить чрезмерные потери тепла. Проконсультируйтесь с поставщиком или производителем каски по поводу подходящих подкладок, которые не снижают защиту, обеспечиваемую каской.
  • Одежда должна быть чистой, так как грязь заполняет воздушные ячейки в волокнах одежды и нарушает ее изоляционные свойства.
  • Одежда должна быть сухой. Перед входом в отапливаемые убежища с одежды следует удалять влагу, удаляя снег.Пока работник отдыхает в нагретом помещении, следует дать возможность поту уйти, открыв шею, пояс, рукава и застежки на щиколотках, или сняв верхнюю одежду. Если зона отдыха достаточно теплая, желательно снять внешний слой (и), чтобы пот мог испариться с одежды.
  • Если тонкая ловкость рук не требуется, следует использовать перчатки при температуре ниже 4 ° C для легкой работы и ниже -7 ° C для умеренной работы. Для работы при температуре ниже -17 ° C следует использовать рукавицы.
  • Хлопок не рекомендуется.Он имеет тенденцию быстро намокать или намокать и теряет свои изоляционные свойства. С другой стороны, шерсть и синтетические волокна сохраняют тепло во влажном состоянии.

Обувь

Ботинки на фетровой подкладке, с резиновым дном и кожаным верхом со съемными войлочными стельками лучше всего подходят для тяжелой работы в холодную погоду, поскольку кожа пористая, позволяющая ботинкам «дышать» и позволять испаряться поту. Кожаные ботинки можно «гидроизолировать» с помощью некоторых продуктов, которые не блокируют поры в коже. Однако, если работа связана с стоянием в воде или слякоти (например,ж., пожаротушение, сельское хозяйство), то необходимо надеть непромокаемую обувь. Хотя они защищают ноги от намокания от холодной воды на рабочем месте, они также предотвращают выход пота. Изоляционные материалы и носки намокают быстрее, чем в кожаной обуви, и увеличивают риск обморожения.

Foot Comfort and Safety at Work содержит общую информацию о том, как выбрать обувь. (Кроме того, примеряя ботинки перед покупкой, наденьте носки того же типа, что и на работе, чтобы обеспечить правильную посадку.)

Носки

Вы можете предпочесть одну пару толстых объемных носков или две пары — один внутренний носок из шелка, нейлона или тонкой шерсти и более толстый внешний носок немного большего размера. Носки с подкладкой из полипропилена сохранят ноги сухими и теплыми, отводя пот от кожи. Однако по мере того, как внешний носок становится демпфирующим, его изоляционные свойства ухудшаются. Если позволяют условия работы, приготовьте запасные носки, чтобы вы могли сушить ноги и менять носки в течение дня. Если надеты две пары носков, внешний носок должен быть большего размера, чтобы внутренний носок не сжимался.

Всегда носите носки подходящей толщины к ботинкам. Если они будут слишком толстыми, ботинки будут «тугими», а носки потеряют большую часть своих изоляционных свойств, когда они будут сжаты внутри ботинка. Стопа также будет «сдавлена», что замедлит кровоток к ступням и увеличит риск холодовых травм. Если носки слишком тонкие, ботинки будут свободно сидеть и могут образоваться волдыри.

Защита лица и глаз

В очень холодных условиях, когда используется защита лица, средства защиты глаз должны быть отделены от носа и рта, чтобы предотвратить запотевание выдыхаемой влаги и обмерзание защитных очков или очков.Выбирайте защитные очки, которые подходят для выполняемой вами работы, а также для защиты от ультрафиолетового излучения солнца, яркого света от снега, метеорита снега / кристаллов льда и сильного ветра при низких температурах.


Психрометрическая диаграмма (обновлена ​​22.07.2014)

Глава 10b: Психрометрическая диаграмма (обновлена ​​22.07.2014)

Глава 10: Смеси воздуха и водяного пара

б) Психрометрическая таблица и Процессы кондиционирования воздуха

Мы заметили из развития в разделе а) , что уравнения, связывающие относительные и удельная влажность, температура (влажный и сухой термометр), давление (воздух, пар) и энтальпия довольно утомительны и неудобны.За это причина Психрометрическая Диаграмма , касающаяся всех соответствующих были разработаны переменные, которые чрезвычайно полезны для проектирования и оценка систем кондиционирования воздуха и градирен.

На первый взгляд психрометрическая диаграмма довольно сбивает с толку, однако с некоторой практикой он становится чрезвычайно полезным инструмент для быстрой оценки процессов кондиционирования воздуха. Большинство популярная широко используемая диаграмма разработана ASHRAE (Американское общество отопления, охлаждения и Инженеры по кондиционированию воздуха), однако мы считаем, что строительство упрощенная версия диаграммы, основанная на приближении различные уравнения могут быть очень полезным инструментом для разработки понимание его использования.Такой подход был предложен Maged. Эль-Шаарави в своей статье «О Психрометрическая диаграмма », опубликованная в ASHRAE Transactions (Paper # 3736, Vol 100, Part 1, 1994) и вдохновил нас на создание следующая упрощенная психрометрическая карта:

Основная информация, используемая для построения диаграммы: данные о насыщении водяным паром (Tsat, Pg), полученные из паровые столы в диапазоне от Tsat = 0,01 ° C до 50 ° C. В удельная влажность ω затем оценивается с использованием относительной влажности φ в качестве параметра для построения различных кривых относительной влажности (синий строк) следующим образом:


где P — стандартное атмосферное давление 101.325 [кПа].

Кривая насыщения (относительная влажность 100%) также кривая точки росы отображается красным линия. Обратите внимание, что на кривой насыщенности влажные и температуры сухого термометра имеют те же значения.

Основное упрощающее допущение в конструкции диаграммы состоит в том, что энтальпия смеси принята равной постоянна на протяжении всего процесса адиабатического насыщения (описанного в Раздел а ). Это означает, что испаряющийся добавленная жидкость не оказывает существенного влияния на энтальпию паровоздушная смесь, обеспечивающая постоянный наклон температуры по влажному термометру / энтальпия (красный) линии определены:


Примечание что на оси ω = 0 (сухой воздух) h = T [° C]

Наконец, удельный объем паровоздушной смеси (зеленые линии) определяется из соотношения идеального газа как


где газовая постоянная R воздух = 0.287 [кДж / кг.K]

Это нормальная практика — отделить перекрывающиеся друг от друга линии энтальпии / температуры по влажному термометру, позволяющие проводить их отдельно оценен. Таким образом, мы вводим наклонную ось энтальпии и энтальпию (черные) линии следующим образом:

Четыре уравнения, выделенные выше, были запрограммированы в MATLAB и используется для построения упрощенных психрометрических диаграмм, показанных над. По ссылке:
MATLAB программа для построения упрощенной психрометрической диаграммы

Превосходный NebGuide (Университет Небраски-Линкольн Публикация расширения), описывающая Как для использования упрощенной психрометрической диаграммы имеет предоставлен Дэвидом Шелтоном и Джеральдом Бодманом.Еще одно полезное руководство предоставлено Марком Картрайтом из Подрядчика Северной Каролины Testing Institute (NCCTI) — это видео на YouTube: Psychrometric Диаграмма упрощенная . Обе направляющие уменьшают путаница, отдельно объясняя 4 из 6 наборов кривых, которые составьте психрометрическую карту. Обязательно просмотрите оба руководства перед продолжение.

Решенная задача 10.1 — Предположим температура наружного воздуха 32 ° C при относительной влажности φ = 60%. Используйте психрометрическую диаграмму, чтобы определить удельную влажность ω [18 г-влаги / кг-воздуха], энтальпия h [78 кДж / кг воздуха], температура по влажному термометру T wb [25.5 ° С], температура точки росы T dp [23 ° C], и удельный объем сухого воздуха v [0,89 м 3 / кг]. Укажите все значения, определенные на диаграмме.

Решенная задача 10.2: Допустим что температура наружного воздуха 8 ° C. Если воздух в комнате 25 ° C при относительной влажности φ = 40%, используйте психрометрическую диаграмму определить, соприкасаются ли окна этой комнаты с снаружи станет туманно.


воздух, соприкасающийся с окнами, станет холоднее, пока не выпадет роса. точка достигнута.Обратите внимание, что в условиях 25 ° C и 40% относительная влажность температура точки росы немного выше 10 ° C, в этот момент водяной пар конденсируется, когда температура приближается к 8 ° C по линии насыщения, и окна будут стать туманным.

______________________________________________________________________________________

Одно из основных применений Психрометрической Диаграмма находится в системе кондиционирования воздуха, и мы обнаруживаем, что большинство людей чувствуют комфортно при температуре от 22 ° C до 27 ° C, а относительная влажность φ от 40% до 60%.Это определяет «комфорт» зона », которая изображена на психрометрической диаграмме, как показано ниже. Таким образом, с помощью таблицы мы либо нагреваем, либо охлаждаем, добавляем влаги или осушите по мере необходимости, чтобы воздух зона комфорта.

Решенная проблема 10.3: Снаружи воздух при температуре 35 ° C и относительной влажности 60% необходимо кондиционировать охлаждение и обогрев, чтобы воздух оставался «комфортным». зона «. Используя Психрометрическую диаграмму, аккуратно нанесите требуемый процесс кондиционирования воздуха и оценка (а) количества влаги удалено [11.5 г-ч30 / кг-сухой воздух], (б) отводимое тепло [(1) — (2), q холодный = 48 кДж / кг-сухой воздух], и (c) количество добавленного тепла [(2) — (3), q тепло = 10 кДж / кг-сухой воздух].

Решенная проблема 10.4: : Горячий сухой воздух при 40 ° C и относительной влажности 10% проходит через испарительный охладитель. Вода добавляется по мере прохождения воздуха через серию фитилей и смесь выходит при 27 ° C. Использование психрометрии диаграмма определяет (а) относительную влажность на выходе [45%], (б) количество добавленной воды [5.4 г-ч30 / кг-сухой воздух], и (c) самая низкая температура, которая могла быть реализована [18,5 ° C].

Охладители этого типа очень популярны в горячих, сухих климата, и широко известно как болото Кулер . Интересное приложение использование болотного кулера для охлаждения питьевой воды в очень горячих окружающая среда описана в публикации Rich Oppel in the At War блог New York Times: ‘Пьянство Из Socks ‘.

Интересное и информативное описание на Психрометрический Схема использования для домашнего скота и теплицы приложения были представлены на веб-сайте PennState Extension Эйлин Э.Фабиан. Другой интересный веб-сайт: Википедия по психрометрии.

Перейти к разделу c) Градирни для паровых электростанций

______________________________________________________________________________________


Инженерная термодинамика, Израиль Уриэли под лицензией Creative Commons Attribution-Noncommercial-Share Alike 3.0 Соединенные Штаты Лицензия

Советы по температуре теста — Пекарня выходного дня

Не «что-то вроде» или «почти»… Я хочу знать точно!

Одна вещь, которую мы узнали во время наших приключений с выпечкой, — это следить за температурой теста.В нашей мини-пекарне температура колеблется от 13ºC / 55ºF зимой до 26ºC / 79ºF и выше летом. Поверьте, эта разница температур сильно влияет на конечный результат вашего хлеба. Когда вы выпекаете много буханок партиями и хотите добиться однородности конечного продукта, вам необходимо контролировать температуру теста.

Немного жесткой теории

Ниже приведена таблица для дрожжей L. sanfranciscensis lactobacillus и C. milleri, которые распространены в закваске.Коммерческие обычные активные дрожжи содержат виды S. cervisiae. Тип C. milleri очень распространен в хлебе на закваске, и он лучше работает при более низких температурах, но S. cervisiae на самом деле достигает пика своей активности примерно при 35 ° C (95 ° F).

Разница в температуре теста в несколько градусов может сильно изменить продолжительность основной ферментации или финальной расстойки. При выпечке хлеба с использованием обычных активных сухих дрожжей оптимальная (только для скорости) температура составляет чуть более 27ºC / 80ºF.


Гораздо жарче и активность дрожжей снижается.При температуре выше 35 ° C / 95 ° F дрожжи фактически бездействуют или мертвы. Бактериальная активность достигает пика при 34 ° C / 93 ° F, поэтому некоторые пекари выбирают ферментацию при 32 ° C / 90 ° F, чтобы получить более кислый хлеб. При 21C / 70F активность дрожжей снижается примерно вдвое, поэтому брожение займет вдвое больше времени.

Правильная температура — самая важная переменная. Скорость роста, например, дрожжей L. sanfranciscensis lactobacillus и C. milleri составляет ln2 / время генерации, то есть скорость роста 0,7 соответствует генерации (время удвоения) около 1 часа.

Если время образования теста при 20ºC / 68ºF составляет 1/2 от времени образования при 30ºC / 86ºF, организм также будет расти вдвое быстрее при 20ºC / 68ºF по сравнению с 30ºC / 86ºF. Итак, важны не абсолютные числа, а отношение скорости роста к скорости роста при оптимальной температуре.

Практические советы по постоянству

Чтобы обеспечить постоянное время брожения и расстойки, мы стремимся к температуре теста около 24ºC / 75ºF. Это температура, которая дает хороший баланс между скоростью и вкусом.Вы можете достичь и поддерживать температуру вашего теста с помощью следующих методов:

  • Купите хороший цифровой термометр; Мы используем термометр CDN DTQ450X ProAccurate Quick-Read и термапен, которые являются отличными, стабильными и быстрыми.
  • Используйте теплую воду зимой или холодную летом, чтобы нагреть тесто до нужной температуры.
  • Температура воды зависит от температуры в вашей комнате и от температуры других ингредиентов.Например, наша Pain Rustique использует 50% -ное предпочтение по отношению к конечному тесту и, таким образом, использует 72% воды в масле. Зимой нам иногда требуется очень горячая вода (до 55 ° C), чтобы получить однородную консистенцию, а новую муку — до 24 ° C. Однако при использовании более низкого отношения предпочтения к конечному тесту, воды с температурой около 30 ° C часто бывает достаточно, чтобы достичь 24 ° C.
  • Старайтесь держать тесто при стабильной температуре. Иногда мы ставим миску с тестом на духовку Rofco, накрытую кухонным полотенцем, и поверх сложенного полотенца.Также отлично подойдет предварительно нагретая и выключенная духовка примерно до 30ºC (используйте термометр для измерения внутренней температуры, так как ваш циферблат на духовке, вероятно, не будет очень полезен).
  • Некоторые люди сами строят расстойный шкаф, используя старый холодильник или кухонный шкаф. С помощью лампочки 40 Вт или 100 Вт в нижней части шкафа (теплый воздух поднимается снизу вверх!) Вы можете легко нагреть внутреннюю часть шкафа. Вы можете использовать простой механический термостат, чтобы включать и выключать лампочку, чтобы контролировать температуру.
  • Если у вас нет средств контролировать температуру окружающей среды для вашего теста, летом вы можете сделать тесто на несколько градусов холоднее, а зимой немного теплее, чтобы компенсировать повышение или понижение температуры.
  • Вода и мука имеют разную теплоемкость. При выпечке хлеба это означает, что влияние воды на температуру теста вдвое больше, чем у муки. Калькулятор теста, который вы найдете в большинстве наших рецептов хлеба, учитывает это.

Хотите узнать больше о расчете правильной температуры воды?
Прочтите эту публикацию в блоге Wild Yeast

.

Добавить комментарий

Ваш адрес email не будет опубликован. Обязательные поля помечены *

*